Chào mừng!

ĐĂNG KÝ THÀNH VIÊN MỚI TẢI ĐƯỢC TÀI LIỆU! Đăng ký ngay!

KHÁCH VÀ THÀNH VIÊN CÓ THỂ TẢI MIỄN PHÍ HƯỚNG DẪN ĐĂNG KÝ THÀNH VIÊN VÀ TẢI » THƯ MỤC MIỄN PHÍYOPOVN
ĐĂNG KÝ NÂNG CẤP THÀNH VIÊN VIP ĐĂNG KÝ NÂNG CẤP THÀNH VIÊN VIP » ĐĂNG KÝ NGAYĐĂNG KÝ NÂNG CẤP THÀNH VIÊN VIP

Yopovn

Ban quản trị Team YOPO
Thành viên BQT
Tham gia
28/1/21
Bài viết
82,025
Điểm
113
tác giả
BỘ 30 ĐỀ THI TUYỂN SINH LỚP 10 MÔN TIẾNG ANH 2023 - 2024 CÓ ĐÁP ÁN được soạn dưới dạng file word gồm 9 trang. Các bạn xem và tải về ở dưới.

ĐỀ THI TUYỂN SINH LỚP 10 MÔN TIẾNG ANH

CÓ ĐÁP ÁN NĂM HỌC 2023 - 2024

ĐỀ 01

A:
PHONETICS ( 1.0 p)

I. Choose the word with different pronunciation from the others. (0.6p)


1. A. hour B. house C. hot D. head

2. A. worked B. wanted C. stopped D. asked

3. A.walks B. cups C.students D. pens

II.Choose the word whose stress pattern is different from that of the others. ( 1.0 pts)

1. A. sewage B. simple C. hobby D. describe

2. A. dancer B. cycling C. balloon D. traffic.

B: VOCABULARY AND GRAMMAR: (5.0pts) I. Choose the best answer. (2.0p)

1.Solar energy doesn’t cause………..

A.pollution B.polluted C.pollute D.pollutant

2. Tet is a festival………… occurs in late January or early February.

A. whom B. when C. where D. which

3. It’s raining. …………….., Mr. Nam has to go to work.

A. Although B. So C. Therefore D. However

4. If I ………….rich, I ……………………….around the world.

A. will be - travel B. am - will travel C. were - would travel D. would be – traveled

5. The girl wishes she………………… in Hue for the festival next week.

A. had stayed B. was staying C. stay D. could stay

6. You don’t like watching this film, …………………?

A. don’t you B. are you C. do you D. did you

7. When he lived in the city, he …………… to the theater twice a week.

A. uses to go B. has gone C. used to go D. was going

8. My house………….. in 1999

A. is built B. was building C. was built D. has been built

9. The entrance examination will be held………………….June 22nd 2012.

A. in B. on C. at D. to

10. Hoa: I suggest going camping next Sunday.- Lan: …………………………..

A. That’s a fine day B. That’s a good idea C. That’s a reason D. That’s a good trip

II. Put the verbs in the brackets into the correct tense or form.(2.0p)

1. The weather is terrible today. If the weather (1.be)……… good, I (2. go).....................for a walk.

2. Yesterday, when we (3.visit)…………………. them, they (4. have)………….. dinner.

3. I (5.write )............ to my pen pal 2 months ago, but I (6.not receive) ……....... his reply since then.

4. We would rather (7. stay)...........................at home than go out on rainy days.

5. I enjoy (8. teach)...................................., but I don't want ( 9. do)......................all my life.

6. My house ( 10. build)………………………………………..at present.

III/ Supply the correct form of the words in brackets to complete the following sentences. (1.0p)

1. The accident happened because he drove …………………………... (care)

2. Air …………….……is one of the problems that people have deal to with. (pollute)

3. Traditional ………………..…... are a good source of fun and entertainment. (celebrate)

4. You should buy this book . It’s very…………………………….. (inform)

5. (Tradition)………….………................……., people eat sticky rice cakes at Tet.

C: READING. ( 2.0 p)

I.
Read the text. Then choose the right sentence A, B, C or D.(1.0p)

Last year, we had a nice holiday. My friend and I went to the seaside for a month. I had been to the seaside several times before, but this was the first time for my friend. Naturally, it was the great event for him. Finally, the day came. It was a fine morning. We got up very early because we wanted to leave home after breakfast. We made the journey by car. We reached the seaside at noon. We spent many hours on the beach. We enjoyed making castles and channels in the sand. People said we ought to spend at least a few weeks at the seaside. If we could stay longer, so much the better.

How long did the writer and his friend spend at the seaside?

A. week B. A few weeks C. A month D. A few months

2. Who went to the seaside the first time?

A. The writer B. The writer and his friend. C. The writer/s family D. The writer/s friend.

3. What was the weather like on the day they started their journey to the seaside?

A. It was bad B. It was rainy C. It was snowy D. It was nice.

4. How did they travel to the seaside?

A. By car B. By train C. By bus D. By air

5. When did they reach the seaside?

A. At 8 o/clock B. At 12 o/clock C. At 4 p.m D. At 9 p.m

II. Read the passage and answer the following questions. (1.0p)

Alexander Fleming was born in 1881 in Scotland. He went to a small school in a village, and when he left school he didn’t go to university. He worked for five years in an office. But his brother, Tom, was a doctor and helped Fleming to go to university and study medicine. So he went to London University and in 1906 he became a doctor. In 1915, Fleming married Sarah McElroy, an Irish woman. They had one son. During the First World War, many soldiers died in hospital because they didn’t have the right medicines. So after the war, Fleming tried to find a drug that could help them. He worked for many years and in 1928 he discovered a new drug and he called it “penicillin”. He later worked with an Australian and a German scientist to develop a drug that doctors could use. In 1945, they won the Nobel Prize in medicine for their work on penicillin.

1. Where was Alexander Fleming born? ………………………………………………………………

2. Did he work in an office before he went to university? ……….………………………….………..

3. What did he study at university? ………………………………………………………….………

4. When did he win the Nobel Prize in medicine? …………………………………………….………

D: WRITING (2.0p)

I. Complete the second sentences without changing the meaning of the first sentences. (1.0p)


1. He has never been late for work.

- Never……………………………………………………………………………………

2. The man is my new boss. He is talking to the lady over there.

- The man …………………………………………………………………………………

3. “Does Mr. Pike live here?” the postman asked the boy.

- The postman asked the boy ……………………………………….…………………….

4. They will build a new school next month.

- A new school……………………………………………………..……………………..

5. Finding an apartment in a big city is not easy.

- It is …...............................................................................................................................

II. Write complete sentences. (1.0p)

1.we /not/ see/ each other/ for/ 5 years.

……………………………………………………………………………………………

2. Mr Bill/ used/ go /swimming/ when/ he/ young.

……………………………………………………………………………………….……

3. She / be / best / student / class.

…………………………………………………………………………………………

4. How long / it / take / you / go / school / everyday?

…………………………………………………………………………………………

ĐÁP ÁN

A:
PHONETICS ( 1.0 p)

I. Choose the word with different pronunciation from the others. (0.6p)


1. A. hour B. house C. hot D. head

2. A. worked B. wanted C. stopped D. asked

3. A.walks B. cups C.students D. pens

II.Choose the word whose stress pattern is different from that of the others. ( 1.0 pts)

1. A. sewage B. simple C. hobby D. describe

2. A. dancer B. cycling C. balloon D. traffic.

B: VOCABULARY AND GRAMMAR: (5.0pts)

I. Choose the best answer. (2.0p)


1.Solar energy doesn’t cause………..

A.pollution B.polluted C.pollute D.pollutant

2. Tet is a festival………… occurs in late January or early February.

A. whom B. when C. where D. which

3. It’s raining. …………….., Mr. Nam has to go to work.

A. Although B. So C. Therefore D. However

4. If I ………….rich, I ……………………….around the world.

A. will be - travel B. am - will travel

C. were - would travel D. would be – traveled

5. The girl wishes she………………… in Hue for the festival next week.

A. had stayed B. was staying C. stay D. could stay

6. You don’t like watching this film, …………………?

A. don’t you B. are you C. do you D. did you

7. When he lived in the city, he …………… to the theater twice a week.

A. uses to go B. has gone C. used to go D. was going

8. My house………….. in 1999

A. is built B. was building C. was built D. has been built

9. The entrance examination will be held………………….June 22nd 2012.

A. in B. on C. at D. to

10. Hoa: I suggest going camping next Sunday. Lan: ………………………………………………..

A. That’s a fine day B. That’s a good idea

C. That’s a reason D. That’s a good trip

II. Put the verbs in the brackets into the correct tense or form.(2.0p)

1. The weather is terrible today. If the weather (1.be)were… good, I (2. go)...would go.....for a walk.

2. Yesterday, when we (3.visit)…visited………. them, they (4. have)…were having…dinner.

3. I (5.write )......wrote....….. to my pen pal 2 months ago, but I (6.not receive) …haven’t received….........….. his reply since then.

4. We would rather (7. stay)........stay...................at home than go out on rainy days.

5. I enjoy (8. teach)...........teaching......., but I don't want ( 9. do)..........to do............all my life.

6. My house ( 10. build)…………is being built……………………………..at present.

III/ Supply the correct form of the words in brackets to complete the following sentences. (1.0p)

1. The accident happened because he drove …………carelessly………………... (care)

2. Air ……pollution………is one of the problems that people have deal to with. (pollute)

3. Traditional ……celebrations………... are a good source of fun and entertainment. (celebrate)

4. You should buy this book . It’s very………informative…………………….. (inform)

5. (Tradition)……Traditionally.............……., people eat sticky rice cakes at Tet.

C: READING. ( 2.0 p)

I.
Read the text. Then choose the right sentence A, B, C or D.(1.0p)

Last year, we had a nice holiday. My friend and I went to the seaside for a month. I had been to the seaside several times before, but this was the first time for my friend. Naturally, it was the great event for him. Finally, the day came. It was a fine morning. We got up very early because we wanted to leave home after breakfast. We made the journey by car. We reached the seaside at noon. We spent many hours on the beach. We enjoyed making castles and channels in the sand. People said we ought to spend at least a few weeks at the seaside. If we could stay longer, so much the better.

How long did the writer and his friend spend at the seaside?

A. week B. A few weeks C. A month D. A few months

2. Who went to the seaside the first time?

A. The writer B. The writer and his friend.

C. The writer/s family D. The writer/s friend.

3. What was the weather like on the day they started their journey to the seaside?

A. It was bad B. It was rainy C. It was snowy D. It was nice.

4. How did they travel to the seaside?

A. By car B. By train C. By bus D. By air

5. When did they reach the seaside?

A. At 8 o/clock B. At 12 o/clock C. At 4 p.m D. At 9 p.m

II. Read the passage and answer the following questions. (1.0p)

Alexander Fleming was born in 1881 in Scotland. He went to a small school in a village, and when he left school he didn’t go to university. He worked for five years in an office. But his brother, Tom, was a doctor and helped Fleming to go to university and study medicine. So he went to London University and in 1906 he became a doctor. In 1915, Fleming married Sarah McElroy, an Irish woman. They had one son. During the First World War, many soldiers died in hospital because they didn’t have the right medicines. So after the war, Fleming tried to find a drug that could help them. He worked for many years and in 1928 he discovered a new drug and he called it “penicillin”. He later worked with an Australian and a German scientist to develop a drug that doctors could use. In 1945, they won the Nobel Prize in medicine for their work on penicillin.

1- (He was born) in Scotland

2- Yes, (he did.)

3- He studied medicine

4- (He won the Nobel Prize in medicine) in 1945.

D: WRITING (2.0p)

I. Complete the second sentences without changing the meaning of the first sentences. (1.0p)


1 - Never has he been late for work.

2 - The man who is talking to the lady over there is my new boss.

3 - The postman asked the boy if Mr Pike lived there

4. A new school will be built next month.

5. It is not easy to find an apartment in a big city

II. Write complete sentences. (1.0p)

1- We haven’t seen each other for 5 years.

2 - Mr Bill used to go swimming when he was young.

3- She is the best student in her class.

4 - How long does it take you to go to school everyday?

ĐỀ 02

A. PHONETICS (1pt)

I. Choose one word whose underlined part is pronounced differently:


1. A. different B. world C. practiced D. disaster

2. A. pollute B. receipt C. species D. accept

3. A. clear B. bear C. hear D. near

II. Choose one word whose stress pattern is different:

4. A. promote B. attract C. trophy D. compete

5. A. develope B. introduce C. discover D. Prepare

B. Vocabulary and grammar (5pts)

I. Choose a,b,c, or d that best completes each unfinished sentence(2pts)


1. You will not succeed .......................... working hard.

a. unless b. without c. if d. although

2. It was raining very ............. so I took my umbrella.

a. wet b. badly c. hard d. firmly

3. I .............. do that if I were you.

a. won’t b. shan’t c. wouldn’t d. don’t

4. You may borrow as many books as you like provided you show them to ............... is at the desk.

a. who b whom c. whoever d. which

5. I wish you ............. stop interrupting me whenever I speak.

a. did b. would c. might d. will

6. I wish I ........ more about the logistics of the expedition.

a. would know b. knew c. know d. can know

7. The little girl ............. when she fell.

a. hurt himself b. hurt herself c. has hurt d. hurt

8. Listen to what I am saying, ............................?

a. don’t you b. do you c. did you d. will you

9. .................. this medicine, and you’ll be well again.

a. Have b. Drink c. Eat d. Take

10. Yoko feels .................. again after her illness but she still cannot work ............. .

a. strong/ hardly b. strongly/ hard c. strongly/ hardly d. strong/ hard

II. Give the correct form of verbs in brackets: (2pts)

1. Listen to these students! What language they (speak) ……………….....?

2. What you (do) ……………………….. If you had a lot of money?

3. Did you advise Jane (join) ....…………………….. in the Vietnamese speaking contest?

4. Who (carry) ……………………..... your bag when you climb Mount Phanxipang?

5. In Viet Nam, it (not snow) ...…………………….. in winter except for Sa Pa and Lang Son.

6. We (always/ make) ....……………………...... to work hard by our parents.

7. Their grandparents prefer (watch) ....…………………… basketball to playing ..……………………... it.

8. My friends (wait) .....…………………….. for me when I got to the stadium.

9. Nothing (do) ........…………………… since he moved here.

10. To avoid (attack) ...……………………... again, the millionaire hired some guards.

III.Give the correct form of the words in the brackets (1pt)

1. Novelists are among the most _____ people in the world. (IMAGINE)

2. Secondary education is _____ and free in many countries. (COMPEL)

3. The song has _____ been selected for the Sea Games 22, Vietnam. (OFFICE)

4. . _____ are doing their best to make people aware of the danger of air pollution. (ENVIRONMENT)

5. Your father is a bit _____. I think he should go on a diet. (WEIGH)

C. Reading(2pts)

I. Read the passage then choose the best answer: (1pt)


Music influences people in different ways or the same person differently at different times. Music may seem to influence people differently. That is because people can react differently to the music. We are able to apply a choosing process to the music we hear. If someone hates jazz, then a jazz piece with a positive effect will probably not make him feel good. A happy song might appear to make an angry person angrier, yet it is not the music itself that is creating the anger; rather it is the positive effect of the music. The angry person does not want to accept the song's happy feeling: it points out his already existing anger, and makes that anger come to the surface. When a piece of music is played and we are listening to it, our body, mind, and feelings are being affected. The musicians of ancient cultures such as China, India, Turkey and Greece understood the effects of music. In fact, Pathagoras, in ancient Greece, introduced a whole science that concerned them. Because the musicians of these ancient cultures understood these effects, they created music that was positive, uplifting, and beneficial. Once the effects of music are better understood, the next step is to gain a better understanding of the music around us, and what effect it is actually having.

1. The text is about_____.

A. The science of music B. Understanding music

C. The effects of music on human feelings D. Music and an angry person

2. Music_____.

A. cannot be chosen B. affects everybody in the same way

C. affects us in different ways D. never make us angry

3. According to the text,_____.

A. Everybody likes jazz B. Jazz always makes us feel better

C. No one likes jazz D. a very angry person sometimes do not accept music

4. In ancient cultures, there used to be a science that concerned the effects of music in_____.

A. China B. India C. Turkey D. Greece

5. The word "Once" has a close meaning to_____.

A. on time B. when C. because D. if

II. Choose the best option to fill in each blank: (1 pt)

Environmental pollution is a term that refers to all the ways by (1)…….man pollutes his surroundings. Man dirties the air with exhaust gases and smoke, contaminates the water with chemicals and other substances, and damages the (2) …..with too many fertilizers and pesticides. Man also pollutes his surroundings in various other ways. For example, people ruin natural beauty by scattering junk and litter on the land and in the water.

Environmental pollution is one of the most serious problems (3) ………….. mankind today. Air, water, and soil are necessary to the survival of living things. Badly polluted air can cause illness, and even death. Polluted water kills fish and other (4) ….life. Pollution of soil reduces the amount of land that is available for growing food. Environmental pollution also brings ugliness (5) …….man’s natural beautiful world.

1. A. that B. whom C. which D. whose

2. A. soil B. forest C. beach D. stream

3. A. faces B. faced C. to face D. facing

4. A. human B. marine C. animal D. plant

5. A. to B. on C. for D. in

IV. Writing:( 2pts) I. Error recognition (1pt)

1. What (A) do you always (B) go to (C) school? By (D) bus.

2. When it was (A) first establish (B) in 1973, it consisted (C) of only two hectares of land (D).

3. If I were (A) her, I'll quit (B) that job immediately (C) and look for another one (D).

4. What (A) do you practise playing (B) the guitar all day fo (C)r? Winning (D) a prize.

5. Vietnam (A) National anthem (B), calling (C) “Tien Quan Ca”, was written (D) in 1954.

II. Rewrite each of the following sentences without changing its meaning :( 1pts)

1. "Please sit down,” the teacher said to me.

=> The teacher asked me ............................................................... .

I fell asleep because the film was boring.

=> The film was so .......................................................................... .

“ What are you doing?”, she asked me.

=> She wondered .................................................................................

4. I don’t find it difficult to get up early in the morning.

=> I am used ....................................................................

5. They have given me two interesting books. => I …………………………………………………….

KEY

A. Phonetics: (1pt)


1. C 2. B 3.B 4. C 5. B

B. Vocabulary and grammar (5pts)

I.

1. b2. c3. c4. c5. b
6. b7. b8. d9. d10. d
II.

1.are they speaking 2. would you do

3. to join 4. will carry

5. doesn't snow 6. are always made

7. watching/ playing 8. were waiting

9. has been done 10. being attacked

III.

1. imaginative 2. compulsory

3. officially 4. Environmentalists 5. overweight

C. Reading(2pts)

I. 1. C 2.D 3.D 4.D 5.B

II. 1.C 2.A 3.D 4.B 5.A

D. Writing:( 2pts)

I.Mistake: 1. A 2. B 3. B 4. D 5.C

II.Rewrite:

1. The teacher asked me to sit down.

2. The film was so boring that I fell asleep.

3. She wondered what I was doing.

4. I am used to getting up early in the morning.

5. I have been given two interesting books.

ĐỀ 03

I.Choose the word whose underlined part is pronounced differently from the others (0,5p).


1. A. thank B. they C. these D. than

2. A. walks B. cups C. students D. pens

3. A. talked B. watched C. lived D. stopped

4. A. stream B. leather C. cream D. dream

5. A. hour B. house C. hot D. head

II. Choose the word or phrase (A, B, C, or D) that best completes each sentence(2ps).

1. We used to ……….past the market on the way to school.

A. walk B. walked C. walking D. have walked

2. Remember to turn …….the light when leaving the office.

A. off B. on C. down D. in

3. Ho Chi Minh City is the ……….one in the country.

A. larger B. large C. largest D. largely

4. Lan is very tired. ________, she has to finish her assignment before going to bed.

A. Although B. So C. Therefore D. However

5. I suggest________some money for poor children.

A. raise B. to raise C. raised D. raising

6. Have you ever ………a letter to the local authorities to complain about the public transport in your town?

A. wrote B. writing C. written D. write

7. When are you ………vacation with your family, Lan?

A. on B. by C. with D. to

8. If you ……late, you wouldn’t finish your work.

A. come B. coming C. to come D. came

9. Huong wishes she ………free time to visit her close friend in hospital.

A. to have B.had C. have D. having

10. I think I’ve lost my new hat. I’ve ………it everywhere but I can’t find it.

A. looked out B. looked in C. looked on D. looked for

III- Identify the underlined word/ phrase (A or B,C,D) that needs correcting to become an exact one. (0,5p)

1. My sister enjoys read about wild animals and natural mysteries.

A B C D

2. Mr. Thach who sing English songs very well is my teacher of English.

A B C D

3. My father asked us not to spending too much time playing computer games.

A B C D

4. Ba can plays the piano better than his friends can.

A B C D

5. I saw the men, the women and the cattle which went to the field

A B C D

IV.Give the correct tense or form of the verbs in brackets(2ps).

a. Don't make noise now.My parents (take) ..(1)............................. their usual afternoon nap

b.While I (have) ..(2)............................. a bath, the telephone (ring)..(3)..................

c. He prefers (play)..(4)............... computer games to (read) ..(5)............... books.

d. Don't forget (lock) ..(6)................ the door before (go) ..(7)............. to bed

e. I (write )..(8)........….. to my penpal 2 months ago, but I (not receive) …(9)…..........…….. his reply since then.

f. If we (not save) …(10)….........…… energy , there will be a shortage in the next century.

V.Read the following passage, then choose the correct answer to questions (1p)

Singapore is an island city of about three million people. It’s a beautiful city with lots of parks and open spaces. It’s also a very (1)______ city.

Most of the people (2)______ in high-rise flats in different parts of the island. The business district is very modern with (3)______ of high new office buildings. Singapore also has some nice older sections. In Chinatown, there (4)______ rows of old shop houses. The government buildings in Singapore are very beautiful and date from the colonial days.

Singapore is famous (5)______ its shops and restaurants. There are many good shopping centers. Most of the goods are duty free. Singapore’s restaurants sell Chinese, Indian, Malay and European food, and the prices are quite reasonable.

1. A. large B. dirty C. small D. clean

2. A. live B. lives C. are living D. lived

3. A. lot B. lots C. many D. much

4. A. is B. will be C. were D. are

5. A. in B. on C. at D. for

VI. Read the passage and answer the following questions. (2ps)

Alexander Fleming was born in 1881 in Scotland. He went to a small school in a village, and when he left school he didn’t go to university. He worked for five years in an office. But his brother, Tom, was a doctor and helped Fleming to go to university and study medicine. So he went to London University and in 1906 he became a doctor. In 1915, Fleming married Sarah McElroy, an Irish woman. They had one son. During the First World War, many soldiers died in hospital because they didn’t have the right medicines. So after the war, Fleming tried to find a drug that could help them. He worked for many years and in 1928 he discovered a new drug and he called it “penicillin”. He later worked with an Australian and a German scientist to develop a drug that doctors could use. In 1945, they won the Nobel Prize in medicine for their work on penicillin.

1. Where was Alexander Fleming born?

……………………………………………………………………………………

Did he work in an office before he went to university?

……………………………………………

3. What did he study at university?

………………………………………………………………………………………

4. When did he win the Nobel Prize in medicine?

………………………………………………………………………………………

VII.Write a new sentence as similar as possible in meaning to the original sentence(2ps)..

1. The girl is very friendly. She lived next door to us.

->The girl ...................................................................................

2.It takes Minh 2 hours to do his homework every day.

->Minh spends ..............................................................................

3.We were late for school because of the heavy rain.

->Because it ....................................................................................

4. My mother used to make us clean the house.

->We used .......................................................................................

5."I'm working in a restaurant, and don't care much for it." she said.

->She said .....................................................................................

6.It’s two years since I last spoke to her.

->I haven’t ...................................................................................

7.Going swimming in the river in the summer is interesting.

->It is..........................................................................................

8.No one in my class is more intelligent than Lan.

->Lan is ...................................................................................

Key

I-Choose the word whose underlined part in pronounced differently from the rest in each group (0.5 đ ) mỗi câu đúng cho 0.1 đ.

1. A​
2. D​
3. B​
4. B​
5. A​
II. Choose the word or phrase (A, B, C, or D) that best completes each sentence(2ps).

(2đ ) mỗi câu đúng cho 0.2 đ


1. A2. A3. C4. D5. D
6. C7. A8. D9. B10. D
III- Identify the underlined word/ phrase (A or B,C,D) that needs correcting to become an exact one. (0,5p)

mỗi câu đúng cho 0.1 đ.


1. B​
2. B​
3. B​
4. A​
5. C​
IV-Give the correct tense or form of the verbs in brackets(2ps).

mỗi câu đúng cho 0.2 đ


1.are taking2. were having3. rang4.playing5.reading
6. to lock7. going8.wrote9. haven't received10. don't save
V.Read the following passage, then choose the correct answer to questions (1p)

mỗi câu đúng cho 0.2 đ


1. D. clean
2. A. live
3. B. lots
4. D. are
5. D. for
VI. Read the passage and answer the following questions. (2ps) mỗi câu đúng cho 0.5 đ

1. (He was born) in Scotland.

2. Yes, (he did.)

3. He studied medicine.

4. (He won the Nobel Prize in medicine) in 1945.

VII.Write a new sentence as similar as possible in meaning to the original sentence,using the words given(2ps).. mỗi câu đúng cho 0.25 đ

1->The girl who lived next door to us is very friendly

2->Minh spends 2 hours doing his homework every day

3->Because it rained heavily, we were late for school

4->We used to be made to clean the house by my mother

5->She said (that) she was working in a restaurant and didn’t care much for it.

6->I haven’t spoken to her for two years

7->It is interesting to go swimming in the river in the summer

8->Lan is the most intelligent (student) in my class

ĐỀ 04

I. Choose the underlined word that has different pronunciation:1điểm


1. a. garbage b. standard c. solar d. lunar

2. a. covered b. installed c. described d. decorated

3. a. chopstick b. charity c. children d. Christmas

4. a. celebration b. plumber c. bulb d. blanket

5. a. energy b. generous c. category d. Suggest

II. Choose one word whose stress pattern is different from the others by circling A, B, C or D. (5 pts)


1. A. medical B. restaurant C. remember D. government

2. A. household B. homework C. garbage D. throughout

3. A. electrical B. interesting C. necessary D. beautifully

4. A. celebrate B. consider C. expression D. criteria

5. A. partner B. between C. visit D. program

III. Choose the best answer: 2 điểm

1. ___ the traffic was bad, we arrived on time.(Although/ In spite of/ Even/ Despite of)

2. I’ll show you around the city when you ___ .(come/ are coming/ will come/ came).

3. If energy ___ inexpensive and unlimited, many things in the world would be different. (is/ will be/ were/ would be).

4. ___ energy can be cheap and clean.(Sun/ Sunny/ Solar/ Solarize).

5. I am writing this to tell you ___ you are missed and loved.(how many/ how much/ how long/ how far).

6. We should learn about keeping the environment ___ .(pollute/ polluting/ polluted/ unpolluted).

7. The pencil ___ I write is made in Japan.(who/ whom/ which/ where).

8. We don’t want ___ like that.(talking/ being talked/ talked/ to be talked).

9. They ___ in Ha Noi since last Monday.(have been/ were/ are/ will be).

10. Let’s go shopping, ___ ?(shall we/ can we/ do we/ should we).

IV. Give the correct form of the word: 1điểm

1. This coffee is too................... to drink. HEAT

2. He drives so ................... that he is sure to have an accident. CARE

3. His parents were ……..................……that he failed the final examinations. DISAPPOINT

4. Do you know what the ............................ of river is ? DEEP

5. He is a very ............................. carpenter. SKILL

6. There are many ---..................------- in our country of a year. CELEBRATE

7. We celebrate our -..................--------- Day on September 2nd. DEPEND

8. You must be -------..................--- when you open that door. CARE

9. We can save ----..................------ resources by using solar energy. NATURE

10. We can’t clean up our --..................-------- rivers and seas over night. POLLUTE

V. Give the correct form of the verb: 2điểm

1. I’m sorry. I ( not finish ) ………………………… this work yet.

2. Poems as well as plays ( write ) ……………………… by Shakespeare.

3. She is having an examination. She wishes she (spend )……………………… a day off on a farm now.

4. We hate ( watch ) ………………………the advertising program on TV.

5. I had a phone call while I ( surf ) ……………………… the web.

6. They ( live ) ………………………in that house since it was rebuilt.

7. I suggest (burn)………………………coal to heat the house.

8. If we (not save)…………………… energy, there (be) ……………… great shortage in the next century.

9. Do you have any difficulty in ( solve ) ……………………… this problem.

VI. Rewrite the sentences: 2điểm

Mary always went swimming after school last year.

---> Mary used to………………………………………………………………...

My daughter has studied English for two years.

---> My daughter started ………………………………………………………...

She wants to buy that villa but it’s too far expensive for her.

---> She wishes ………………………………………………………………….

It took Nam two days to have a trip to the countryside last week.

---> Nam spent …………………………………………………………………..

They have changed the date of the meeting.

---> The date …………………………………………………………………….

Finding an apartment in a big city is not easy.

---> It is not ……………………………………………………………………..

We enjoy playing soccer.

---> We are interested …………………………………………………………..

8. “Do your parents still live in Nha Trang now?” said Nam to me.​

Nam asked me……………………………………………………………………​

VII. Read the following passage and choose the item A,B,C or D that best answer each question about it. (1.0ps)

Nick Johnson lives with his parents, and his sister. They live in Wembley, in north London. Nick’s mum is called Sue. She works in a supermarket. His dad is called Jim and he works in a bank. Nick’s sister is called Tracy. She is nine years old. There is also a dog in the family. His name’s Fred.

Where does Nick’ family live?

A. America B. England C. Scotland D. Australia

Which does the word “his” in line 4 refer to?

A. Nick’s mother B. Nick’s sister C. Nisk’s cousin D. Nick’s dog

What’s Nick’s sister’s name?

A. Sue B. Jim C. Tracy D. Fred

Who is Jim?

A. Nick’s father B. Nick’s mother C. Nick’s cousin D. Nick’s sister

Which of the following is not TRUE?

A. Nick’s family live in London. B. Nick’s father works in a bank.

C. Nick’s mother works in a supermarket. D. Nick’s sister is five years old.

VIII. Read the passage and answer the following questions . (1p)

Jeans are the most popular kind of clothing in the world . They are popular almost eveywhere, in France, in China, Italy and Vietnam. Most of people love to wear Jeans . Rich people, poor people , young people and even some old people wear them . In the United States , they are the only kind of traditional clothes. In other countries, young people wear them because they want to look modern . Jeans are the sign of young people . Everyone wants to be modern, young and independent.

Are jeans the most popular kind of clothing in the world?

............................................................................................................................

Where are jeans popular?

.............................................................................................................................

Do rich people love to wear jeans?

................................................................................................................................

What are the only kind of traditional clothes in the United States?

..................................................................................................................................

Why do young people in other countries wear them?

....................................................................................................................................

IX. Make sentences using the cues: (1.5 m)

1. If / I / you / , / I / learn harder.

............................................................................................................................................

2. She / suggest / go / Huong pagoda / this weekend.

.......................................................................................................................................................

3. Two days ago, / my brother / come / school late / because / he / get up / late.

......................................................................................................................................................

4. The Internet / wonderful invention / modern life.

......................................................................................................................................................

5. Our school / going / hold / English-speaking contest / celebrate / Teacher’s Day.

......................................................................................................................................................

Key for FP4

I. Choose the underlined word that has different pronunciation:

1. a. garbage

2. d. decorated

3 d. Christmas

4. b. plumber

5. c. category

II. Choose one word whose stress pattern is different from the others by circling A, B, C or D.

1. C 2. D 3. A 4. A 5. B

III. Choose the best answer:

1. Although

2. come

3 were

4. Solar.

5. how much

6. unpolluted.

7. which

8. to be talked

9. have been

10. shall we

IV. Give the correct form of the word:

1. celebrations

2. depended

3. careful

4. natural

5. polluted

V. Give the correct form of the verb:

1. haven’t finished

2. were

3. spent

4. watching

5. was surfing

6. have lived

7.burning

8. don’t save / will be

9. solving

VI. Rewrite the sentences:

1. Mary used to go swimming after school last year.

2. My daughter started studying English two years ago.

3. She wishes that villa were cheap / weren’t expensive

4. Nam spent two days having a trip to the countryside last week

5. The date of the meeting has been changed by them

6. It is not easy to find an apartment in a big city

7. We are interested in playing soccer.

8. Nam asked me if my parents still lived in Nha Trang then

VI. Read the following passage and choose the item A,B,C or D that best answer each question about it.

1.B 2.D 3.C 4.A 5.D

VIII. Read the passage and answer the following questions .

1.Yes, they are

2.They are popular almost everywhere, (in France, in China,Italy and Viet Nam)

3.Yes, they do

4.They are jeans

5.Because they want to look modern



ĐỀ 05

I. Choose the word whose underlined part is pronounced differently from the others.


1. A. claimed B. warned C. occurred D. existed

2. A. health B. appear C. ready D. heavy

3. A. tidal B. sight C. mineral D. describe

4. A. though B. throw C. through D. thought

5. A. thunder B. erupt C. trust D. pull

II. Choose the best answer from the four options given to complete each sentence.

6. I can’t understand the French visitors. I wish I________French.

A. knew B. will know C. know D. have known

7. The church________about 100 years ago.

A. is built B. was built C. will be built D. has been built

8. People in Israel are going to celebrate their festival________is called Passover.

A. whose B.who C. which D. where

9. ________I was really tired , I couldn’t sleep.

A. Even though B. So C. Therefore D. Because of

10. Lan is very tired. ________, she has to finish her assignment before going to bed.

A. Although B. So C. Therefore D. However

11. I suggest________some money for poor children.

A. raise B. to raise C. raised D. raising

12. She asked me if I________a laptop computer the following day.

A.buy B. will buy C. bought D. would buy

13. I ________telephone her if I knew her number.

A. would B. have to C. will D. shall

14. Honda motorbikes________in Viet Nam.

A. produce B. will produce

C. are produced D. would be produced

15. We have learnt English________2001.

A. for B. since C. in D. during

16. We________already________Huong Pagoda.

A. were / seeing B. Have / seen

C. are / seeing D. Will / see

17. All the houses in the area________immediately.

A. has to rebuilt B. had to rebuild

C. have to be rebuilt D. have to rebuild

18. If he________soon, he might miss the train.

A. isn’t coming B. doesn’t come

C. won’t come D. didn’t come

19. Mr. Long said that he________in Ho Chi Minh City.

A. lived B. is living C. has lived D. will live

20. Your sister works in a foreign company, ________she?

A. isn’t B. didn’t C. wasn’t D. doesn’t

III. Identify the underlined word/ phrase (A or B,C,D) that needs correcting to become an exact one.

21. My sister enjoys read about wild animals and natural mysteries.

A B C D

22. Mr. Thach who sing English songs very well is my teacher of English.

A B C D

23. My father asked us not to spending too much time playing computer games.

A B C D

24. Ba can play the piano better more than his friends can.

A B C D

25. We can save nature resources by using solar energy.

A B C D

IV. Choose the correct answer from the options below, then complete the following passage,

Today, supermarkets are found in almost every large city in the world. But the first supermarket (26)_________opened only fifty years ago. It was opened in New York by a man named Michael Cullen. A supermarket is different (27)___________other types of stores in several ways. In supermarkets, goods are placed on open shelves. The(28)__________ choose what they want and take them to the checkout counter. This means that fewer shop assistants are needed than in other stores. The way products are displayed is another difference between supermarkets and many other types of stores; (29)__________ example, in supermarkets, there is usually a display of small inexpensive items just in front of the checkout counter: candies, chocolates, magazines, cheap foods and so on. Most customers (30)__________go to a supermarket buy goods from a shopping list. They know exactly what they need to buy. They do the shopping according to a plan.

26. A. is B. has been C. was D. were

27. A. in B. from C. of D. with

28. A. customers B. managers C. assistants D. sellers

29. A. in B. for C. of D. by

30. A. who B. what C. which D. whom

V. Complete the second sentence so that it has a similar meaning to the first one.

31. They have just sold that old house.

=> That old house .....................................................................

32. In spite of the bad weather, they had a wonderful holiday.

=> Although ..........................................................................................................................

They will build a new mall here.

=> A new mall ........................................................................

34. Unless he takes these pills, he won’t be better.

=> If ......................................................................................................................................

35. Despite working hard, he can’t support his large family.

=> Although ..........................................................................................................................

VI. Complete each of the following sentences, using the words given.

36. If/ it/ not rain/ tomorrow/ I/ go/ camping/ my friends.

=> ..........................................................................................................................................

37. If/ I/ meet/ alien/ outer space/ I/ invite/ home/ talk.

=> ..........................................................................................................................................

I/ never/ travelled/ by/air.

=> .......................................................................................................

39. The students/ study/ for exam/ now. => ...............................................................................................

40. It/ be/ such/ cold day/ we/ decide/ not/ go out.

=> ..........................................................................................................................................

41. If / I / you / , / I / learn harder. =>…………......... ...............................................................

42. She / suggest / go / Huong pagoda / this weekend.. => …….......................................................................

43. Two days ago, / my brother / come / school late / because / he / get up / late.

........................................................................................................................................................

44. The Internet / wonderful invention / modern life.

...........................................................................................................................................................

45. Our school / going / hold / English-speaking contest / celebrate / Teacher’s Day.

Key to FP5

1. D 2. B 3. C 4. A 5. D 6. A 7. B 8. C 9. A 10. D

11. D 12. D 13. A 14. C 15. B 16. B 17. C 18. B 19. A 20. D

21. B read => reading 22. B sing => sings 23. B spending => spend

24. C more than => than 25. B nature => natural

26. C 27. B 28. A 29. B 30. A

31. That old house has just been sold (by them).

32. Although the weather was bad, they had a wonderful holiday.

33. A new mall will be built here (by them).

34. If he does not take these pills, he won’t be better.

35. Although he works hard, he can’t support his large family.

36. If it does not rain tomorrow, I will go camping with my friends.

37. If I met an Alien from outer space, I would invite him/her/it to my home and talk/ to talk.

38. I have never travelled by air.

39. The students are studying for their exam now.

40. It was such a cold day that we decided not to go out.

ĐỀ 06

I
/(1,0p Choose the word in each group that has the underlined part pronounced differently from the rest

1. A. nothing B. cover C. morning D. done

2. A. how B. town C. slow D. power

3. A. change B. chemistry C. choose D. cheese

4. A. match B. catch C. math D. watch

5. A. bread B. great C. break D. steak

II/ (3,0 points) Choose the best word or phrase to complete the following sentences.

1. I like reading books ______ tell about different peoples and their cultures.

A. who B. whom C. which D. where

2. “Buy me a dictionary on your way back,______?”

A. will you B. don’t you C. can’t you D. are you

3. Mary usually walks to school but today she ______ a bike.

A. ride B. rides C. riding D. is riding

4. She worked hard ______she passed the exam.

A. so B. if C. because D. as

5. I’ll give you a lift to the station if you _______ in a hurry.

A. were B. are C. will be D. would be

6. I’m feeling pretty tired. Do you mind ______ me home?

A. taking B. took C. take D. to take

7. Nam looks much _______ today than yesterday.

A. good B. well C. better D. the best

8. Scientists and engineers have invented devices to remove ______ from industrial wastes.

A. polluted B. pollute C. polluting D. pollutants

9. The dress you bought is of very good quality. It ______ very expensive.

A. may be B. must be C. should have been D. must have been

10. The Pikes put ______ their trip because of bad weather.

A. away B. up C. out D. off

11. Neither Linh nor her classmates ______ the National Museum so far.

A. visit B. visited C. have visited D. has visited

12. Nga: “You look nervous! ______” – Hoa: “This thunder scares me to death.”

A. How are you? B. Why’s that? C. Come on! D. What’s wrong?

13. He was homesick and he ______ all his family and friends.

A. missed B. wished C. desired D. hoped

14. I'm very ______ to you for putting in so much hard work.

A. thoughtful B. grateful C. considerate D. careful

15. If you come to the theatre late, you have to wait until the ______ to get in.

A. break B. refreshment C. interval D. half-time

III. Choose the underlined part (marked A, B, C or D) that needs correction.

21. I look forward to have the resolution to the problem I have mentioned.

A B C D

22. They have moved nothing in your room while they sent you to the hospital.

A B C D

23. Aren’t you afraid that they will sack you if you didn’t start coming to work on time?

A B C D

24. I had to drive to the factory to pick up my brother, who’s car wouldn’t start.

A B C D

25. It was not easy for us getting tickets for the concert.

A B C D

IV. Give the correct tense or form of the verbs in brackets.

26. When my mother came home from work, I_________(cook) dinner.

27. Jane_________(go) to school by bicycle every day.

28. The children_________(play) badminton in the stadium now.

29. You_________(go) to London last month?

30. The candidates mustn’t_________(bring) books into the examination room.

V/ (1,5 points) Read the passage and choose the best option A, B, C or D to answer the question.

Along with jogging and swimming, cycling is one of the best all-round forms of exercise. It can help to increase your strength and energy, giving you more efficient muscles and a stronger heart. But increasing your strength is not the only advantage of cycling. Because you’re not carrying the weight of your body on your feet, it’s a good form of exercise for people with painful feet or backs. However, with all forms of exercise it’s important to start slowly and build up gently. Doing too much too quickly can damage muscles that aren’t used to working. If you have any doubts about taking up cycling for health reasons, talk to your doctor and ask for his/her advice.

Ideally you should be cycling at least two or three times a week. For the exercise to be doing you good, you should get a little out of breath. Don’t worry that if you begin to lose your breath, it could be dangerous and there must be something wrong with your heart. This is simply not true; shortness of breath shows that the exercise is having the right effect. However, if you find you are in pain then you should stop and take a rest.

1. People with back problems might go cycling because__________.

A. it enables them to carry the weight of their body on their feet

B. it does not make them carry the weight of their body on their feet

C. it helps to make their backs become stronger

D. it helps them to relieve their backache

2. All forms of exercise must be started__________.

A. gradually B. quickly C. strenuously D. violently

3. According to the writer, it is best to go cycling__________.

A. once a week B. at least two or three times a week

C. at least two or three times a day D. every day

4. You should not worry about the shortness of breath because__________.

A. it shows that there is something wrong with your heart

B. it shows that you should stop and take a rest

C. it is a sign of exercise having the right effect

D. it is a sign of your getting rid of your heart problem

5. Which of the following is NOT included in the advantages of cycling?

A. Giving you a stronger heart B. Increasing you strength and energy

C. Giving you better muscles D. Making you look younger

VI/ (1,5 points) Fill in each blank with one suitable word.

Energy is one of the problems that many people are interested in. It is not an unfamiliar word. It is heard, said, discussed day after day. It is close to everyone’s (1)_____ life. You turn on the lamp and it is (2)______ that gives you light. You turn on a TV and it is energy that gives you pictures and sound. You ride a motorcycle and it is energy that gives you movement. You (3)______ your meals and it is energy that gives you heat to boil rice.

The problem is that the demand for energy is rising and that the price of energy is getting (4)______ and higher. The supply of energy on earth is limited. It cannot provide us all forever. The shortage of energy in the future is inevitable. Therefore, (5)______ energy is a must if we want to continue to live in a safe and sound world.

VII/ (1,5p) Complete the following sentences, using the words or phrases suggested.

1. I / feel / hopeful / that / we / find / suitable house / very soon.

2. She / not / play / piano / well / as / her sister.

3. I wish / could / give you / all the expensive things / life.

4. You / hear / Maria / since / you / leave school?

5. The people / live / next door to us/ keep / have / all night parties.

VIII/(1,5p) Finish each of the following sentences in such a way that it means exactly the same as the sentence printed before it.

1. People are going to build a new library in the area.

à A new library _______________________________________________.

2. “Why don’t you take a day off tomorrow?” she said to me.

à She suggested that ____________________________________________.

3. The weather is bad so we can’t go camping.

à If the weather ________________________________________________.

4. He was punished because he behaved badly.

à Because of __________________________________________________.

5. The keeper had no sooner opened the case door than the lion attacked him.

àHardly ______________________________________________________.

Key to FP6

I
/ (1,0 điểm) Mỗi đáp án chọn đúng được 0,2 điểm.

* Lưu ý: Thí sinh chỉ cần viết đáp án A, B, C hoặc D

1. C. morning2. C. slow3. B. chemistry4. D. watch5. A. bread
II/ (3,0 điểm) Mỗi đáp án chọn đúng được 0,2 điểm.

* Lưu ý: Thí sinh chỉ cần viết đáp án A, B, C hoặc D

1. C. which2. A. will you3. D. is riding4. A. so5. B. are
6. A. taking7. C. better8. D. pollutants9. B. must be10. D. off
11. C. have visited12. D. What’s wrong?13. A. missed14. B. grateful15. C. interval
III. Choose the underlined part (marked A, B, C or D) that needs correction.

21. I look forward to have the resolution to the problem I have mentioned. having

A B C D

22. They have moved nothing in your room while they sent you to the hospital. since

A B C D

23. Aren’t you afraid that they will sack you if you didn’t start coming to work on time? don’t

A B C D

24. I had to drive to the factory to pick up my brother, who’s car wouldn’t start. whose

A B C D

25. It was not easy for us getting tickets for the concert. to get

VI. Give the correct tense or form of the verbs in brackets.

26. When my mother came home from work, I_________(cook) dinner. was cooking

27. Jane_________(go) to school by bicycle every day. goes

28. The children_________(play) badminton in the stadium now. are playing

29. You_________(go) to London last month? Did you go

30. The candidates mustn’t_________(bring) books into the examination room. bring

V/ (1,5 điểm) Mỗi đáp án chọn đúng được 0,3 điểm.

1. B2. A3. B4. C5. D
VI/ (1,5 điểm) Mỗi đáp án đúng được 0,3 điểm.

1. daily2. energy3. cook/ prepare4. higher5. saving
VII/ (1,5 điểm) Mỗi câu viết đúng được 0,3 điểm.

1. I feel hopeful that // we will/(can) find // a suitable house very soon.

2. She doesn’t/(can’t) play // the piano // as well as her sister.

3. I wish I // could give you all the expensive things // in life.

4. Have you // heard from Maria // since you left school?

5. The people who/(that) // live next door to us // keep (on) having all night parties.

The people living // next door to us // keep (on) having all night parties.

* Trong từng câu viết, nếu thí sinh có sai sót phần nào, Giám khảo trừ điểm thành phần của phần đó.

VIII/
(1,5 điểm) Mỗi câu viết đúng được 0,3 điểm.

1. A new library is // going to be // built in the area.

2. She suggested that I // (should) take a day off // the following day/(the next day).

3. If the weather weren’t // bad, we // could go camping.

If the weather were // fine/(nice/ better), we // could go camping.

4. Because of his bad // behavior //, he was punished.

Because of behaving // badly, // he was punished.

5. Hardly had // the keeper opened the case door when // the lion attacked him.



ĐỀ 07

I. Choose the word which has the underlined part is pronounced differently from the others (1pt)


A. question B. pollution C. nation D. information

A. carried B. looked C. managed D. opened

A. energy B. terrify C. destroy D. second

A. sight B. considerate C. divide D. tidal

A. control B. pagoda C. remote D. move

II. Choose the best answer A, B, C or D to complete each of the following sentences (2pts)

They suggest ........... more trees in the school yard.

A. plant B. planted C. to plant D. planting

Her son felt sleepy………….. he went to bed.

A. however B. so C. although D. because of

They ………………. her since she left school.

A. haven’t seen B. aren’t seen C. doesn’t see D. don’t see

She used to……………….. soccer in the afternoon two years ago.

A. played B. playing C. play D. plays

I wish my home………………. in the village.

A. was B. were C. is D. will be

…………….. Mrs.Thoa was tired, she helped her children with their homework.

A. If B. And C. Or D. Although

They arrived in Ho Chi Minh city………….………..Monday morning.

A. in B. on C. a D. to

I thanked the man………………. helped me move the fridge.

A. whom B. who C. whose D. which

You don’t like this film,………………?

A. do you B. don’t you C. are you D. doesn’t you

If I were you, I…………………….ask the doctor for advice.

A. can B. will C. must D. would

III. Supply the correct form of the words in brackets to complete the following sentences. (1pt)

The film is so…………that I have seen it twice. (INTEREST)

Air ………….. is one of the problems that people have deal to with. (POLLUTE)

My teacher teaches ………….. sciences. (NATURE)

The gas from the chemical factory was ……………... (HARM)

She got the………… from the library. (INFORM)

III. Choose the best underlined word or phrase in each sentence that needs correcting.(1pt)

I enjoy to collect stamps in my free time .

A B C D

Her teacher encouraged her taking part in the competition.

A B C D

If he knew her address, I will write to her.

A B C D

The man asked us not play near the lake.

A B C D

The exercises were such difficult that I couldn’t do them.

A B C D

V. Put the verbs in the brackets into the correct tense forms. (1pt)

I think we should (give)…………….. poor children books, notebooks and school things.

I (not/meet) ……………. Nam since he left school.

Nga (take) …………… many trips to Ho Chi Minh City since 2005.

If Trang had an alarm clock, she (arrive) …………… to school on time .

What will you do if you (decide) ……………. to take a space trip?

VI. Read the text, then answer the questions below it (2 pts)

Clothes can tell a lot about person. Some people like very colorful clothes because they want everyone to look at them and they want to be the center of things. Others like to wear nice clothes, but their clothes are not colorful or fancy. They do not like people to look at them. Clothes today are very different from the clothes of the 1800s. One difference is the way they look. For example, in 1800s, all women wore dress. The dresses all had long skirts. But today, women do not wear dresses with long skirts. Sometimes they wear short skirts. Sometimes they wear pants. Another difference between 1800s and today is the cloth. In the 1800s, clothes were made only from natural kinds of cloth. They were made from cotton, wool, silk. But today, there are many kinds of man-made cloth. A lot of clothes are now made from nylon, rayon or polyester.

Why do some people like very colorful clothes?

g...................................................................................................................................................

What kind of clothes do other people like to wear?

g...................................................................................................................................................

Were the clothes of the 1800s the same as today ?

g...................................................................................................................................................

What do sometimes women wear today?

g...................................................................................................................................................

Are there many kinds of man-made cloth now?

g...................................................................................................................................................

VII. Complete the second sentence so that it has the same meaning with the first one. (2pts)

He didn’t go to the party because he was busy.

He was busy, …………………………………………………………

It’s a pity. We can’t speak English well.

We wish ……………………………………………………………...

“Do you like classical music ?” She asked me.

She asked me ……………………………………………...................

The man is my new boss. He is talking to the lady over there.

The man ……………………………………………………………..

“Why don’t you buy them this picture as a wedding present, Tom ?”

Jane suggested ………………………………………………………

41. What a pity my wife can't speak English.

g I wish ...................................................................................................................................................

42. The man called the police. His wallet was stolen.

g The man ..............................................................................................................................................

43. English people speak very quickly. That's why I can't understand them.

g If ..........................................................................................................................................................

44. The last time we saw Peter was on Sunday.

g We haven't.............................................................

45. This city had old buildings, but none can be found nowadays.

g There used .........................................................................................................................................

ĐỀ 08

I. Choose the word underlined part is pronounced differently from that of the others:


1. A. seatB. headC. meatD. feed
2. A. watches B. washesC. clausesD. likes
3. A. pleasedB. smokedC. stoppedD. missed
4. A. nuclearB. humorC. polluteD. tunic
5. A. earthB. gatherC. ethnicD. think
II. Choose the best answer (A,B,C or D) to complete the sentences:

6. The final examination will be held ___ June 12th, 2007.

A. inB. onC. atD. to
7. How many languages are there ___ the world?

A. inB. onC. overD. through
8. Egypt is famous ___ ancient pyramid.

A. onB. toC. fromD. for
9. The boy looks very proud ___ his success at school.

A. onB. ofC. atD. in
10. When the trucks leave the police, the ground is covered ___ trash.

A. inB. byC. withD. of
11. You have read this article on the website,________?

A. don't youB. aren't youC. didn't youD. haven't you
12. All the students are looking forward ___ their summer vacation in the countryside.

A. to spendB. spendC. to spendingD. spending
13. Did your children enjoy ___ in the sea?

A. swimmingB. swimC. to swimD. swam
14. "When is Mr Ba planning to retire?

- He ___ for the company for thirty years now".

A. worksB. is workingC. will workD. has been working
15. Flowers should ___ in warm places.

A. be keepB. keptC. be keptD. be keeping
16. They went on walking ___ it started to rain.

A. thoughB. becauseC. butD. despite
17. Study harder ___ the exam.

A. if you will failB. and you would failC. unless you failD. or you will fail
18. The girl wishes she ___ in Hue for the festival next week.

A. had stayedB. was stayingC. stayD. could stay
19. Miss Lan stopped ___ her letter because she had to leave for the hospital.

A. to writeB. writeC. writtingD. wrote
20. Do you know the man ___ car Jack is driving?

A. whoB. whomC. whichD. whose
21. "I suggest going to Vung Tau on the weekend." - "___".

A. That's a fine dayB. That's a good ideaC. That's a tripD. That's a reason
22. "___" - "Don't worry. I can fix it".

A. Are you worried?B. May i fix your TV?C. The TV is out of order again.D. I want to fix your TV.
III .Choose the word or phrase (A, B, C or D) that best fits the blank space in the following passage:

New Year is one of the most important (23) ___ in the United States. On New Year's Eve, most people go to the parties. At twelve o'clock (24) ___ night, everyone says "Happy New Year" and they (25) ___ their friends and relatives good luck. New Year's Eve is usually a long night to this holiday children (26) ___ as witches, ghosts or others. Most children go from house to house asking for candy or fruit. (27) ___ the people at the house do not give (28) ___ candy, the children will (29) ___ a trick on them. But this (30) ___ ever happens. Many people give them candy or fruit.

23. A. festivalsB. meetingsC. contestsD. courses
24. A. onB. atC. inD. for
25. A. dreamB. greetC. wishD. congratulate
26. A. wearB. dressC. put onD. take off
27. A. WhetherB. SoC. AlthoughD. If
28. A. theyB. themC. theirD. theirs
29. A. sayB. tellC. playD. speak
30.A. hardlyB. hardC. soonD. always
IV. Choose the underlined word or phrase (A, B, C or D) that needs correcting:

31. You're tired although you stayed up too late to watch TV last night.
A B C D

32. Tuan sighed sad when he heard that he failed the test.
A B C D

33. Don't speak English with him; he hardly not know any English.
A B C D

34. 90 percent of earthquakes occurs around the Pacific Rim.
A B C D

V. Use the correct form of the word given in each sentence:

35. Fuel ________ can be cut down by having fewer cars on the roads. (consume)

36. Many chemicals have a ________ effect on the environment. (disaster)

37. ________, the typhoon didn't cause any damage on the village. (lucky)

38. The next stage in the ________ of television is interactive TV. (develop)

VI. Read the passage and then write True or False in your answer sheet:

Daddy,

I am writing this to tell you how much you are missed and loved. I will always remember that day - my wedding day. You were standing there with tears in your eyes while I was walking towards my groom. You gave me a hug, and the feeling that you never wanted to let me go. But at last I had to leave you and start my new life... a moment in time that lasted forever. I now have children, Dad, but I will always be your little girl! Happy Father's Day.

39. This is a letter written by a daughter to her father.

40. The father in the letter is not loved much.

41. The man did not let his daughter go at last.

42. The writer of this letter hasn't got any children.

VII. Rewrite the sentences so that they are nearest in meaning to the sentence printed before them:

43. Ba said, "I am going to meet your sister in front of the station, Liz".
- Ba told Liz that …………………………………………………………………

44. The last time she went out with him was two years ago.
- She hasn't ………………………………………………………………………

45. She is not on board now, so she can't experience these marvelous things.
- If she ……………………………………………………………………………

46. The show was interesting to the boys.
- The boys felt ……………………………………………………………………

ĐỀ 09

I. Choose the word that has the underlined part pronounced differently. (0,6m)


1. A. son B. lemon C. bacon D. iron

2. A. imagine B. magazine C. discipline D. Valentine

3. A. pipe B. bill C. side D. smile

II. Choose the word that its main stress is placed differently from the others. (0,4m)

1. A. tutor B. highlight C. lunar D. enroll

2. A. selection B. national C. recycle D. convenient

III. Choose the best option. (3m)

1. The Internet is a very fast to get ----------

A. inform B. information C. informatic D. informative

2. A meteor is also called a shooting star or …………. star

A. flying B. falling C.fallen D.dropping

3. If they are pupils, they……… wear uniform.

A. must B. would C. could D. had

4. She sings very-------

A. beauty B. beautify C. beautiful D. beautifully

5. ---------- the manager is out today, I’ll sign the letter.

A. Because B. Because of C. As D. A and C

6. Can you turn------- the radio ? I’m learning my lessons

A.on B. in C. off D. for

7. I live in Da Lat , ------- is one of the most beautiful cities of Vietnam

A.who B. which C. whom D. whose

8. My friends come to stay ------ us ------ Christmas.

A. to/ at B. with / in C. to / in D.with / at

9. If he ------ you , he would do this work.

A. is B. were C. had D. be

10. Environmental............is a serious problem facing mankind today.

A. pollution B. polluted C.pollute D. polluting

11. I am late, ..................?

A. am I B. am not I C. aren’t I D. not am I.

12. She said they............there the following day.

A. were B. come C. will be D. would be

13. They .......... a new hospital in my neiborhood now.

A. are building B. were building C. building D. are being built

14. He has ……………. interesting book that we all want to borrow it.

A. such B. such a C. such an D. so

15. In my family lighting ____for 20% of the electricity bill.

A. wastes B. collects C. saves D. accounts

IV. Read the passage, choose True (T) or False (F), then answer these questions (1,5 points)

Garbage is a serious problem in the United States. Every year, American produce 308 billion pounds of garbage (about 140 billion kilos)! A lot of garbage goes into landfills. However, many landfills do not have enough room for all the garbage.

One answer to the garbage problem is recycling. Half of all the garbage in the United States can be recycled and used again. Recycling helps the landfill problem and saves enough energy, too. When people recycle newspapers, they save trees. When they recycle glass, plastic, or cans, they save oil and electricity. Recycling is good for everyone.

1) True(T) or False (F)

a) Garbage isn’t a serious problem in the United States. -------

b) We save trees if we recycle glass. -------

c) 50% garbage in the United States can be recycle and used again. -------

2) Answer the questions

a) Do many landfills have enough room for all the garbage?

………………………………………………………………………………………………

b) What do people save when they recycle glass, plastic, or cans?

………………………………………………………………………………………………

V. Read the passage and do the exercise.(1,5m)

It’s not enough that the computer is invading our work and play world .It has started to invade our shopping world as well. Shopping by computer, or teleshopping, is a phenomenon that is beginning to appear in homes, stores, hotels, and even airports. The service allows the shopper to look at electronic catalogs and to order items, such as dishes, clothing or concert tickets, without leaving the computer. This modern way of shopping is convenient, specially for busy people.

1. The computer is now invading ………

A. our work and play world B. our work, play and shopping world

C. our shopping world D. our business world

2. Shopping by computer is also called ………

A. teleshopping B. window shopping

C. shopping mall D. duty-free shopping

3. The service allows the shopper to ………

A. see the good B. make a bargain

C. shop on line D. buy low-priced goods

4. Internet shopping is convenient for …………

A. people who don’t have much free time B. people who have to travel a lot

C. people who do the housework D. people who don’t like shopping

5. According to the passage, you can’t do your teleshopping without ………

A.a television B. a shopping list

C. a telephone D. a computer

VI. Complete the sentences using the cues(1.5m)

1. Why don’t you install a burglar alarm in your house?

- I suggest you .......................................................................................................................

2. We watched the football match between Liverpool and Manchester United last Saturday.

- The football match ..............................................................................................................

3. Tom failed the exam because of his laziness.

- Because Tom ......................................................................................................................

4. Does your brother use the Internet every day?

- Your brother uses ...............................................................................................................?

5. “Do your grandparents still live in Nha Trang now?” said Nam to me.

- Nam asked ........................................................................................................................

VII. Make the sentences using the cues given(1,5m)

1/ Last weekend / my family / two day trip / home village./ We / start / the journey / very early / morning.

………………………………………………………………………………………………

2/ After three hours / travel / bus / we / reach / old banyan tree / entrance / the village.

………………………………………………………………………………………………

3/ Every one / feel tired / take a rest / the tree.

………………………………………………………………………………………………

4/ After that / we start/ walk / the village / twenty minutes / come / my grandparents’ house.

………………………………………………………………………………………………

5/ We / spend / enjoyable weekend / the country./We / feel / happy and healthy/ the trip.

………………………………………………………………………………………………

The end

ĐỀ 10

Question I. (1,0p)

A . Circle the word whose underlined part is pronounced differently from that of the others


1. A. region B. regard C. logical D. energy

2. A. stopped B. laughed C. walked D. stayed

3 .A .televisions B. doors C. lights D. windows

B . Find the word A, B, C, or D which has different stress pattern in line


4. A. exist B. avoid C. support D. notice

5 .A.hungry B. disease C. spacious D. danger

Question II. Choose the best words or phrase (A,B,C or D) to complete the sentences (2.ps)

1. Hanoi is famous …………….its beauty.

A . for B . about C . with D. by

2. Let’s go out and enjoy the sunshine, …………..? – That’s a good idea.

A . will you B . don’t you C . won’t we D . shall we

3. He’s used…………… in heavy traffic everyday.

A . drive B . to drive C . driving D . to driving

4. If the weather …………tomorrow, we’ll have a picnic.

A . is B . were C . was D. will be

5. Walking is a good form of exercise for……….. young ………. old.

A . neither / or B . either / nor C . both / and D . both / or

6. The New Year is celebrated ………..midnight …….. January 1.

A . in / on B . at / on C. at / in D . in /in

7. Can you …….. the volume a little? I can’t hear it very clearly.

A . play B . sing C . down D .turn

8. He is man ……….. we wanted to find.

A . who B . whose C . whom D . which

9.A “They suggest going to Sapa on the weekend.”

B “………………………….”

A . That’s a fine day B . That’s a good idea C . That’s a trip D . That’s a reason

10. ………… Mai is very tired,she has to finish her homework before she goes to bed.

A. Eventhough B. So C. However D.Therefore

Question III : Give the correct tense of the verbs in the brackets (1 pts):

I. ( write ) .............. to my pen pal 2 months a go, but I ( not receive )................. his reply since then.

I can’t go out because it ( rain ).............. and I ( not have )........................ a rain coat.

She ( say ) .......she would phone me this morning , but it’s now 12.30 and she ( not phone ) ..... yet.

He had a bad fall while he ( repair ).............................. his roof.

Laura wishes that she ( not live ).......................... in a small flat .

6 . Our school ( repair) ...............................in some years

7 Nga suggests ( help)............................... the poor

Question IV. Give the correct tense or form of the words in brackets . (1,0 p )

1.. We often take part in many ……………..activities at school. (culture)

2 The Internet has helped students study more……………… (effective)

3. There are many………..in Viet Nam every year ( celebrate)

4
Solar energy does not cause ……………. (pollute)

5 …………….
is one of big proplems in the world ( forest)

Question V :Choose the underlined word or phrase that needs correcting: (1pt)


1. Would you like me helping you with your homework ?

2. Fish must keep in a refrigerator or it will spoil.
3. Tom is always forget his keys and that really annoys me.
4. She spends her free time to visit galleries and museums.
5 . My father prefers watching films at home than going to the cinema .

Question VI. Read the passage and circle the best answer to each of the following questions about it (1,0p)

Almost a hundred thousand people were killed and half a million homes destroyed as a result of an earthquake in Tokyo in 1923. The earthquake began a minute before the noon when the inhabitants of Tokyo were cooking their midday meals. Thousands of stoves were overturned as soon as the earth began to shake. As a result, small fires broke out everywhere and quickly spread. The fire engines were prevented from going to help because many of the roads had cracked open. It was impossible to use fire fighting equipment as most of the water pipes had burst. Consequently, over ninety percent of the damage caused by fire rather than by the collapse of the buildings.

How many people were killed in the earthquake in Tokyo in 1923?

A. Ten thousand people B. Almost a hundred thousand people

C. Thousands of people D. Many people

When did the earthquake begin?

A. began a minute before the noon in 1923 B. began a minute before the noon in 1924

C. began a minute before the noon in 1925 C. began a minute before the noon in 1922

What happened as soon as the earth began to shake?

A. people died B. the water pipes had burst

C. Thousands of stoves were overturned D. the roads had cracked open

Why was the fire engines prevented from going to help?

A. because the water pipes had burst B. because no one helped

C. because small fires broke out everywhere D. because many of the roads had cracked open

What was most of the damage caused by?

A. fire B. the roads C. water pipes D. the collapse of the buildings

Question VII Read the following passage, then choose the correct answer (1p)

Jeans are very popular with (1)________people all over the world. Some people say that jeans are the “uniform” of youth. But they haven’t always been popular. The story of jeans (2)________almost two hundred years ago. People in Genoa, Italy made pants. The cloth made in genoa was (3)________“jeanos”. The pants were called “jeans”. In 1850, a saleman in California began selling pants made of canvas. His name was Levi Strauss. Because they were so strong, “Levi’s pants” became (4)________with gold miners, farmers and cowboys. Six years later Levis began making his pants with blue cotton cloth called denim. Soon after, factory (5)_______in the US and Europe began wearing jeans. Young people usually didn’t wear them.

1. A. rich B. old C. young D. poor

2. A. start B. starts C. was starting D. started

3. A. call B. calls C. calling D. called

4. A. famous B. popular C. good D. wonderful

5. A. workers B. drivers C. cowboys D. Farmers

SECTION D : WRITING (2ps)

Question VIII. Complete the second sentences without changing the meaning of the first sentences. (1.0p)

I don’t have a map, so I can’t show you the way

=> If I…………………………………………

The man is my new boss. He is talking to the lady over there.

=> The man ………………………

“Does Mr. Pike live here?” the postman asked the boy.

=> The postman asked ……………………….

4. They will build a new school next month.

=> A new school……………………..……………………..

5 Though he lives in England, he can’t speak English well.

=>In……………………………………..

Question IX. Write sentences with the cues given. (1 p)

He / used / wear / uniforms / when / he / be / school.

………………………………….

2. I/not/ meet / parents / since last Sunday.

………………………………………………….

3. They / say / they / be / back / following day.

……………................................................

4. She / wish / she / have / new computer/ next school year

….........................................................

5 I / meet / Lan, / mother/ be / kind to everyone………………….........................................................

KEY

Question
Content​
Mark
Question I
(1p)
1.B. regard
2.D. stayed
3.C. lights
4. D notice
5. B. disease
0,2
0,2
0,2
0,2
0,2
0,2
Question III
(1,0p)
wrote
haven’t received
is raining
don’t have
said
hasn’t phoned
was repairing
didn’t live
will be repaired
helping
0,1
0,1
0,1
0,1
0,1
0,1
0,1
0,1
0,1
0,1
Question IV
(1p)



1 Cultural
2 effectively
3 celebrations
4 pollution
5 Deforestation
0,2
0,2
0,2
0,2
0,2
Question V
(1p)
1:C
2:A
3 : B
4: C
5 : D
0,2
0,2
0,2
0,2
0,2
Question VI
(1p)
1B
2 A
3 C
4 D
5 A
0,2
0,2
0,2
0,2
0,2

Question
VII
(1p)

1-C
2-D
3-D
4-B
5-A
0,2
0,2
0,2
0,2
0,2
Question VIII
(1p)

VII.
1 - If I had a map, I could show you the way.
2 - The man who is talking to the lady overthere is my new boss.
3 - The postman asked the boy if Mr Pike lived there.
4 - A new school will be built next month.
5- In spite of living in England, he can’t speak English well

0,2
0,2

0,2
0,2
0,2
QuestionIX
(1ps)

1. He used to wear uniforms when he were at school.
2. I haven't met my parents since last Sunday.
3.They said they would be back the following day.(the next day)
4. She wishes she would have a new computer next school year
5 I met Lan , whose mother is kind to everyone
0,2
0,2
0,2
0,2
0,2

ĐỀ 11
I Pronunciation A. Circle the word underlined part is pronounced differently from that of . (1.0 point)

1. A. mention B. option C. federation D. question
2. A. ghost B. office C. long D. modern
3. A. waited B. mended C. objected D. faced
B Find the word A, B, C, or D which has different stress pattern in line

A.energy B.plentiful C. disappear D.celebrate
A.recycle B. description C. contribute D atmosphere
II. Circle the best answer to complete each of the following questions. (3.0 points)
1.The book ______ consists of four parts has been translated into Vietnamese.
A. which B. what C. who D. whose
2. My friend is fond of _______ to music in the evening.
A. listen B. listening C. to listen D. listened
3. My sister loves watching the stars………….. night.
A. on B. in C. at D. for
4. He was born ______ September 10th, 1996.
A. in B. on C. at D. from
5. Traveling by air is not cheap and it isn’t interesting ______.
A. either B. neither C. too D. also
6. “I suggest going to the beach on the weekends.” “____________”.
A. That’s a fine day! B. That’s a good idea! C. That’s a good trip! D. That’s a reason!
7. If I were you, I _______ to buy that car.
A. would forget B. will forget C. forget D. forgot
8. Lan studied hard, ______ she completed her exam successfully.
A. although B. so C. because D. since
9. “I can’t find my wallet, Tom” – “Don’t worry. I’ll help you to ______ it”
A. look for B. take care of C. put on D. turn off
10. _______ he wasn’t invited, he went to the party.
A. Although B. Because C. So D. As
11. His house looks very large and beautiful. It is _________house.
A. a seven-room B. a seven-rooms C. seven room D. seven rooms
12. "What would you like to drink?" - "_________".
A. Yes, please B. Milk, please C. No, thanks D. OK
13. Lan is very tired. _________, she has to finish her assignment before going to bed.
A. Although B. So C. Therefore D. However
14. Your sister works in a foreign company, …………..she?
A. isn’t B. didn’t C. wasn’t D. doesn’t
15. It's very hot today. I wish I_________on the beach now.
A. am B. was C. were D. have been
III. Complete each sentence with appropriate forms of the verb in brackets (2.0 points)
1. Hung (1.know)………. a little English, so he (2.wish) ……....he (3.can speak) ………it fluently.
2.When I entered his room, I ( 4.see ) …………… him (5.sleep)…………….. in a chair
3.My sister is on her summer holiday. She ( 6. lie ) ………………on the beach at the moment .
4.If I ( 7. be ) ……………..you , I would buy that house
5. I enjoy (8. teach)………….., and I (9. teach)……………………. for ten years.
6. I suggest ( 10. put ) ……………….garbage bins around the schoolyard.
IV. Read the passage and circle the best answer to each of the following questions about it. (1.0 point)

Last year, we had a nice holiday. My friend and I went to the seaside for a month. I had been to the seaside several times before, but this was the first time for my friend. Naturally, it was a great event for him. Finally, the day came. It was a fine morning. We got up very early because we wanted to leave home after breakfast. We made the journey by car. We reached the seaside at noon. We spent many hours on the beach. We enjoyed making castles and channels in the sand. People said we ought to spend at least a few weeks at the seaside. It we could stay longer, so much the better.

How long did the writer and his friend spend at the seaside?

A. A week B. A few weeks C. A month D. A few months

2. Who went to the seaside the first time?

A. The writer B. The writer and his friend

C. The writer’s family D. The writer’s friend

3. What was the weather like on the day they started their journey to the seaside?

A. It was bad B. It was rainy C. It was snowy D. It was nice

4. How did they travel to the seaside?

A. By car B. By train C. By bus D. By air

5. When did they reach the seaside?

A. At 8 o’clock B. At 12 o’clock C. At 4 p.m D. At 9 p.m

V. Choose the best word from the box to fill in each gap in the following passage. (1.0 point)

too
from
have
to live
destroying is
We are all slowly (1) …destroying………… the earth. The sea and the rivers are (2) ……………… dirty to swim in. There (3) ……………so much smoke in the air that it is unhealthy (4) …………………….in many of the world’s cities. In one well- known city, for example, the gases (5)………… cars pollute the air so much that traffic policemen (6) …………..to wear oxygen masks.

VI. Finish each of the following sentences in such a way that it means exactly the same as the sentence printed before it. (1.6 points)

1. " Do you like pop music?" he asked me.

-> He asked me if ..............................................................................

2. The flat's very noisy, but we enjoy living there.

-> Even though........................................................................

3. I am sorry that I can't come to your party.

-> I wish I ............................................................................

4. Mary is reading an English novel. It is interesting.

-> The English novel which ……………………………………………

5. They were late for school because it rained heavily.

-> It rained .............................................................................................................

6. Work hard or your parents will be unhappy.

® If you……………………………………….

7. They grow rice in tropical countries

® Rice is……………………………………

8. It takes Minh 2 hours to do his homework every day.

® Minh spends ......................................................................

VII : Complete each of the following sentences, using the words given. (0.4 point)

1. If/ I/ meet/ alien/ outer space/ I/ invite/ home/ talk.

...........................................................................................................................................

2. mom/ never/ travelled/ by/ air/ before.

...........................................................................................................................................

Key to FR9
I. Circle the word whose underlined part is pronounced differently from that of the others.

(Total: 1 point; 0.2 point for each correct answer)


1. D
2. A
3. D
4. C
5. D
II. Circle the best answer to complete each of the following questions.

(Total: 3 points; 0.2 point for each correct answer)


1. A
4. B
7. A
10. A
13. C
2. B
5. A
8. B
11. A
14. D
3. C
6. B
9. A
12. B
15. C
III. Complete each sentence with appropriate forms of the verb in brackets.

(Total: 2.0 points; 0.2 point for each correct answer)


1. knows6. is lying
2. wishes7. were
3. could speak8. teaching
4. saw9. have taught
5. sleeping10.putting
IV. Read the passage and circle the best answer to each of the following questions about it.

(Total: 1 point; 0.2 point for each correct answer)


1. C
2. D
3. D
4. A
5. B
V. Choose the best word from the box to fill in each gap in the following passage.

(Total: 1 point; 0.2 point for each correct answer)


1. destroying
2.too
3. is
4. to live
5.from 6 have
VI. Finish each of the following sentences in such a way that it means exactly the same as the sentence printed before it.

(Total: 1,6points; 0.2 point for each correct answer)


1. He asked me if I liked pop music.

2 Even though the flat's/is very noisy, we enjoy living there.

3. I wish I could come to your party.

4. The English novel which Mary is reading is interesting.

5. It rained heavily, so they were late for school.

6.If you don’t work hard, your parents won’t be happy

If you work hard, your parents will be happy

7.Rice is grown in tropical countries

8Minh spends 2 hours doing his homework everyday

VII Complete each of the following sentences, using the words given.

(Total: 0,4points; 0.2 point for each correct answer)


1. If I met an alien from outer space, I would invite him/her/it to my home and talk/ to talk.

2. I have never travelled by air(before).



ĐỀ 12

I. Circle the word whose underlined part is pronounced differently from the others.


1. .a.home b. hour c.hand d.hear

2. a. school b. match c. teach d. chair

3. a. protects b. kicks c. misses d. stops

4. a. needed b. stopped c. missed d. talked

5. a. kind b. pink c. nice d. pine

II: Choose a word whose stress pattern is different from the rest of the group.

1.a.Enjoy b.collect c.escape d.answer

2.a.summer b.birthday c.busy d. machine

3.a. forget b. repeat c. allow c. morning

4.a. raincoat b. bedroom c. hot- dog d. short- sighted

5. a.unimportant b. underground c. encourage d. imperfect
III. Choose the word or phrase ( A,B,C or D) that best fits the blank space in each sentence.

1. The entrance examination will be held………………….June 22nd 2012.

A. in B. on C. at D. to

2. You don’t like watching this film, …………………?

A. don’t you B. are you C. do you D. did you

3.The bus collected us at 5 o’clock early............. the morning.

A.on B.from C.in D.at

4. They went on working……………….. it started to rain.

A. though B. because C. but D. in spite of

5. The girl wishes she………………… in Hue for the festival next week.

A. had stayed B. was staying C. stay D. could stay

6. We used to………………….. fishing when we were young.

A. went B. gone C. go D. going

7. Do you know the man………………… met our teacher yesterday?

A. who B. whom C. which D. whose

8. If I ……………….. time, I’ll write to you.

A. had B. have C. had D. will have

9. My school_________in 1990 by worker.

A. is built B. was building C. was built D. has been built

10. My father asked us_________too much time playing computer games.

A. not to spending B. did not spend C. not to spend D. to not spent

11. If I were a flower, I_________a sunflower.

A. was B. were C. will be D. would be

12. He didn't run...................................to get there first.

A. enough fast B. fast enough C. enough to fast D. to fast enough

13. ..........................the weather was bad, the football match was put off.

A. though B. Because of C. Because D. So

14.Even....................it was very cold, Mr Ba was waiting for the bus.

A. although B. though C. despite of D. in spite of

15. September 2nd has been considered a great holiday in Viet Nam...................1945.

A. since B. on C. for D. from

IV. Give the correct tense or form of the verbs in brackets.

1.The students (learn) English in the classroom at the moment.

..............................................................................................................................................

2. The house (destroy) by the storm last week.

..............................................................................................................................................

3. When I (come) yesterday, they (have) dinner.

..............................................................................................................................................

4. If I were you, I (not spend ) much time on video games.

..............................................................................................................................................

5. Would you mind (close) the window?

..............................................................................................................................................

6.Laura wishes that she (not live ) in a small flat .

..............................................................................................................................................

7.They (repair) our school some years ago.

..............................................................................................................................................

8. Nga suggests ( help)the poor

..............................................................................................................................................

9. My mother came in while I (watch) TV in my room.

..............................................................................................................................................

10.It is easy (find) a beautiful house in the big city.

..............................................................................................................................................

V. Complete each sentence with the correct form of the word given in the brackets:

1. Fish can’t live in this lake because the water is ………….....…. (pollute)

2. I saw your school’s ……………...........… (advertise)

3. Energy-saving bulbs should be ……..........……….. (use)

4. Passover of the Jewish people is a festival which celebrates ……. from slavery. (free)

5. ………………, people eat sticky rice cakes at Tet. (tradition)

6. English is fun, so we are............................................ in learning it. (interest)

7.The Internet is a wonderful............................................... of modern life. (invent)

8.The price of ......................................... has gone up again. (electric)

9.You must be........................................... when you open that door. (care)

10.We can protect the environment by .................................... air pollution. (reduce)

VI.choose the underlined word or phrase in each sentence that need correcting.

1. I saw the men, the women and the cattle which went to the field.

A B C D

2. Look at the two dictionarys and you will see they are the same in some ways.

A B C D

3. There is no water in the house. If there is, we could cook dinner.

A B C D

4. She did her test careful last week.

A B C D

5. Would you mind turn on the lights, please? It’s too dark for me to read.

A B C D

6. Yesterday when Jane phoned, Mike had dinner with his family.

A B C D

7. This book was writing by Jack London, an American writer.

A B C D

8. Mr. Johnson used to running every morning, but he doesn’t now.

A B C D

9. Bob and Susan usually goes out for a meal on Saturday evenings.

A B C D

10. I am afraid this camera is too expensive for we to buy.

A B C D

VII. Read the passage.

Tet holiday is celebrated on the first day of the Lunar New Year in Viet Nam. Some weeks before the New Year, the Vietnamese clean their houses and paint the walls. New clothes are bought for the occasion. One or two days before the festival, people make Banh chung, which is the traditional cake, and kinds of jam. On the New Year’s Eve, the whole family get together for the reunion dinner. Every member of the family should be present during the dinner in which many different kinds of dishes are served. On New Year morning, the young members of the family pay their respects to the elders. And the children receive lucky money wrapped in red tiny envelopes. Then people go to visit the neighbors, friends and relatives.

a,True or false.

1.Vietnamese people clean their houses some weeks before the New Year.

2.A week before Tet, people make Chung cake.

3.The young members of the family pay their respects to elders.

4.The elders receive lucky money wrapped in red tiny envelopes.

b,Answer the questions.

When is Tet holiday celebrated in Viet Nam?

…………………………………………………………………………………………

2.Do they make Banh chung three days before the festival?

…………………………………………………………………………………………

3.What do they do on the New Year’s Eve?

…………………………………………………………………………………………

4.Who receives lucky money?

…………………………………………………………………………………………

5.Do they visit their neighbors, friends, relatives on New Year morning?

…………………………………………………………………………………

VIII. Rewrite each sentence, beginning as shown, so that the meaning stays the same.


1. They did the work excellently.

The work………………………………………..............…………………………………

2. “I am going to meet my pen pal in front of the station”, said Lan.

Lan said that……………………………………..........…………………………………..

3. Mai can sing better than Hoa.

Hoa can’t………………………..................………………………………………………

4. I don’t have a motorbike.

I wish………………………………...................................................…………………….

5. Finding an apartment in a big city is not easy.

It is not..................................................................................................................................

6. My sister started learning English 5 years ago.

My sister has .......................................................................................................................

7.It takes Long two hours to do his homework every day.

Long spends..........................................................................................................................

8. “Do you go to school on Sunday?” my friend asked.

My friend asked...................................................................................................................

9. Mary usually went swimming after school last year.

Mary used.............................................................................................................................

10. They live near a lake. The lake is heavy polluted. (use a relative pronoun to combine the sentence.)

..............................................................................................................................................

IX. Use the words or phrases given to complete the sentences

1.
He / be/ fond of / listen /music

……………………………………………………………………………………………

2 . She / be / best / student / class.

……………………………………………………………………………………………

3 .How long / it / take / you / go / school / everyday?

…………………………………………………………………………………………

4. I/ not finish/ homework / yet.

……………………………………………………………………………………………

5. Lan / start/ learn/ English/ 6 / years/ ago

……………………………………………………………………………………………

6.I / look / forward / to / see / you / again.

………………………………………………………………………………………

7. If / I / you / , / I / learn harder.

...........................................................................................................

8. She / suggest / go / Huong pagoda / this weekend.

…………………………………………………………………………………………

9. They / suggest / we / protect / environment.

……………………………………………………………………………………………

10. Unless/ weather/ nice/ we/ stay/ home.

……………………………………………………………………………………………

ĐỀ 13

I. Choose the underlined part that is pronounced different from others. (1pt)


1. A. invited
2 .A. house
3. A. cartoon
4. A. motion
5.A.mean
B. attended
B. harm B. carry B. question B. ready
C. celebrated
C. hour C. face
C. mention C. head
D. splayed
D. husband D. coin
D. collect
D. fiction
D. weather
II. Choose the best answer. (2pt)

1. You and I went there together ,………….?

A. didn’t you B. didn’t I C. didn’t we D. did we

2. John isn’t going to go, and Peter isn’t………….

A. too B. either C. also D. as well

3.We wish you…………here with us now.

A. be B. were C. are D. would be

4.I was born………….. 1987.

A. on B. in C. of D. at

5.If he ………soon, he will miss the train.

A. doesn’t come B. isn’t coming C. didn’t come D. won’t come

6 We are too late. The plane………….off ten minutes ago.

A. took B. has taken C.had taken D. was taken

7.John didn’t do his homework, ………………….?

A. didn’t he B. did John C. didn’t John D. did he

8. Lam passed the test successfully…………………he worked very hard.

A. though B. so C. If D. because

9. If you explained it to him more slowly, he ……………….. it.

A. understands B. would understand C. understood D. will understand.

10. People are now interested in saving money and……………………..resources.

A. natural B. naturalize C. nature D. naturally

III. Supply the correct verb form. ( 2pt)

1.They (1 move )………………………..three times since they got married.

2 We ( visit )………………………… Ha Long Bay next week.

3.If the weather is fine, we (3 go ) ……………………..camping next Sunday.

4.I wish I ( 4 can )…………………….go out for dinner with you now.

5.While I ( 5 stand )……………………. in front of the school gate, I (6 see) ………….…..Nam.

6.Mrs Thanh suggests ( 7 take )…………………….showers to save water.

7.At 6.30 last night, when I (8 have) ………………………dinner, my friend phoned me.

IV: Read the passage and circle the best answer A, B, C, D to complete the passage. (1pt)

David Evans is a farmer. He does farming on his own land in Wales. The farm has belonged to his family for generations. David (1)………..sheep on his land. He does not employ (2) ………fulltime workers. His sons helps him when they are not (3) …………school. The guests from the city enjoy their quiet holidays in the clean country (4) ……….and they come to visit their farm. However, it’s a lonely life for the Evans family when the guests have gone. The questions is that if his children want to (5)………….the farm when Mr. Evans give up working.

A. feeds B. grows C. raises D. leads

A. some B. more C. much D. any

A. in B. for C. at D. on

wind B. sight C. life D. air

take off B. take over C. take after D. take care

V.Read the passage then anwer the questions.(1pt)

Millions of pound worth of damage has been caused by a storm which passed across the north of England last night. The River Ribble burst its banks after heavy rain.

People were rescued from the floods by the firemen, who received numerous calls for help. Wind speeds reached ninety miles an hour in some places.Roads were blocked by fallen trees and electricity lines were brought down, leaving thousands of homes wihtout electricity. “ Everything possible is being done to get the situation back to normal,” a spokesman said.

1. What has been caused by a storm which passed across the north of England?

……………………………………………………………………………………

2. When did the storm happen?

…………………………………………………………………………………...

3. What happened to the River Ribble?

……………………………………………………………………………………

4.Who rescued many people from the floods?

…………………………………………………………………………………..

5. Did wind peeds reach 90 miles an hour in some places?

………………………………………………………………………………………

VI. Identify the underlined word/ phrase (A or B,C,D) that needs correcting to become an exact one. (1pt)

1. The picture was painting by Michael last year.

A B C D

2. There’s the woman who she sold me the handbag.

A B C D

3. Mr. Smith is going to buy a new Japanese car, doesn’t he?

A B C D

4. I met a lot of interesting people while I was studying at Ho Chi Minh City.

A B C D

5. If I were you, I didn’t buy that expensive car.

A B C D

VII.Rewrite the following sentences so that they are nearest meaning to the sentence printed before.(2p)

1. It takes Hoa 2 hours to do her homework every day.

Hoa spends.............................................................................................................

2. We were late for school because of the heavy rain.

Because it………………………………………………………………………...

3.I started living here two years ago.

I have ……………………………………………………

4.People speak English as the first language in Australia.

English …………………………………………………..

5. I’m sorry I don’t know the way to your house.

I wish ....................................................................................................................

6.She doesn’t work hard, so she doesn’t get good marks.

If …………………………………………………………….

7. "I'm working in a restaurant "she said

She said…………………………………………………………………………….

8. The girl was injured in the accident. She is now in the hospital.

The girl……………………………………………………………………………

Key

I.Câu 1: 1 điểm: Mỗi ý đúng được 0,2 điểm

1D 2C 3C 4B 5A

II. 2 điểm .Mỗi ý đúng 0,2

1.C 2.B 3.B 4.B 5.A 6.A 7.D 8.D 9.B 10.A

III.2 điểm .Mỗi ý đúng 0,25

1. have moved 2.are visiting 3.will go 4.could 5.was standing 6.saw 7.taking 8.was having

IV.1 điểm. Mỗi ý đúng 0,2.

1.C 2.D 3.C 4.D 5. C

V . 1 điểm. Mỗi ý đúng 0,2.

1.Millions of pound worth of damage has been caused by the storm.(which pass across the north of England.)

2. It happened last night.

3. The River Ribble/It burst its banks .(after heavy rain)

4. The firemen did./The firemen rescued many people from the flood.

5. Yes, it did.

VI. 1 điểm .Mỗi ý đúng 0,25.

1. B 2.C 3.D 4. D 5.C

VII. 2 điểm. Mỗi ý đúng 0,25.

1.Hoa spends 2 hours doing her home work everyday

2.Because it rained heavily, we were late for school.

3.I have lived here for two years.

4.English is spoken as the first language in Australia.

5.I wish I knew the way to your house.

6.If she worked hard, she would get good marks.

7.She said she was working in a restaurant.

8 The girl who is now in the hospital was injured in the accident.

ĐỀ 14

I. Circle the word whose underlined part is pronounced differently from the others.( 1 pt
)

1. A. action B. national C. partial D. question

2. A. tidal B. sight C. describe D. cinema

3. A. connect B. comfort C. computer D. contest

4. A. cough B. though C. rough D. tough

5. A. pleased B. erased C. increased D. amused

II. Choose the word or phrase ( A,B,C or D) that best fits the blank space in each sentence. ( 2pts)


1. - Mary: “Is 11 o’clock alright?” - Tom: “______.”

A. Yes, that’s fine B. No, I don’t want to go C. Wait a minute D. Sounds interesting

2. Typhoons, floods or droughts can easily ______ a harvest.

A. finish B. destroy C. provide D. defeat

3. You should pay ______ to what the instruction is saying.

A. attendance B. intention C. convention D. attention

4. My friends usually enjoy my ______ of humor.

A. sense B. kind C. means D. form

5. - Mother: “Could you do me a favor?” - Kate: “______.”

A. No, thanks. I’m fine B. Yes, that’s kind of you C. Yes, sure D. Yes, thank you

6. Practising ______ English anywhere you can is a way to better your speaking skill.

A. speak B. speaking C. to speak D. for speaking

7. Soak some old newspapers in a ______ of water overnight.

A. pair B. bunch C. bucket D. tube

8. Mrs. Thanh: “______ you give the book to me, please?” - Mai: “Of course.” .

A. Will B. Shall C. Do D. Should

9. The winners are the first ones ______ a fire.

A. make B. makes C. making D. to make

10. Do you know the man………………… met our teacher yesterday?

A. who B. whom C. which D. whose

III. Give the correct tense or form of the verbs in brackets.( 2pts)

1. How many times you (1.be) to London this year?
2. The two girls (2.play) chess over there are my classmates.
3. We must take an umbrella. It (3.rain).
4. The guests (4.arrive) while I (5.still / cook).
5. The rice-cooking festival (6.hold) every two years.
6. I often (7.climb) trees when I was a little girl.
7. Two miles (8.be) enough for her to go jogging every morning.
8. I’m very bored. My mother always (9.complain) about her housework.
9. Fred would like (10.admit) to the college.
IV. Choose the word or phrase ( A,B,C or D) that best fits the blank space in the passage. ( 1,5pts)

Have you ever been to Britain? I’ve dreamed of going there (1) ______ I was a little girl and finally, in the spring of 2012, it happened. I was there three weeks and (3) ______ much of that time soaking up the culture, history, and scenery of this incredible island. I visited several districts in England, Wales and Scotland and even (4) ______ a site trip to Ireland. Every place I went to seemed to be more interesting and more beautiful than the last.

The tour I was with began in London. Of all the sights I saw there, the Tower of London was the most (6) ______. The Tower is a building complex of incredibly rich history.

Also on the Thames are the Houses of Parliament and, of course, the clock tower housing “Big Ben”. I saw so many landmarks I can hardly remember them all. Although it was just a glimpse of London I did, (7) ______, get a chance to see all the most famous places.

After a couple of days in London we traveled to the south of England stopping to visit Stonehenge. If you don’t know about Stonehenge there are a couple of links on my links page that will (10) ______ you to sites that can fill you in.

1. A. for B. while C. since D. because

2. A. spent B. took C. did D. used

3. A. did B. got C. spent D. made

4. A. interesting B. interested C. interest D. interestingly

5. A. at first B. at all C. at least D. at that

6. A. reach B. take C. make D. send

V. Read the passage and answer the questions.(1pt)

Every child in England must receive full time education from the age of five to sixteen. Many parents send their children to state schools where the tuition and the equipment, textbooks and exercise books are free. About 5% of children go to fee-paying school. Under the old system, children attended primary school (from five to eleven) which was followed by a secondary school course (from eleven to fifteen or over. At eleven every child had to sit for the eleven –plus examination. It consisted of tests in English, Arithmetic and intelligence tests. Those with the highest marks (about 20% of the children) went to grammar school. The others went to technical school.

Questions:

When do English children start and finish their full time education?

…………………………………………………………………………………

Which schools do many parents send their children to?

………………………………………………………………………

Do they have to buy textbooks and exercise books at state schools?

………………………………………………………………………………

What did every child have to do at eleven under the old system?

………………………………………………………………………

Who were allowed to go to grammar school?

………………………………………………………………………………

VI. Rewrite each sentence, beginning as shown, so that the meaning stays the same.(1,5pts)

1. I sent my friend a letter in London last week.

ð A letter _____________________________________________.

2. I last saw him when I was a student.

ð I haven’t _________________________________________________.

3. “Don’t forget to bring your passports with you when you go abroad.”

ð She reminded ______________________________________________.

4. Smoking is not allowed in the ward.

ð Would you __________________________________________________?

5. He failed the exam because of his laziness.

ð Because he__________________________________________________.

VII. Use the words or phrases given to complete the sentences.(1pt)

1. Instead / reuse / plastic bags / we / use / cloth bags.ð_________________________________________________.

2. Will / you / have / look / house / me / while / I / be / holiday?

ð_________________________________________________.

3. We / hope / give / more / green / color / town / and / earn / money / group.

ð _____________________________________________________.

4. “Friends of the Earth” / be / organization / protect / planet.

ð _______________________________________________________.

5. It / brave / him / jump / river / save / child.

ð_______________________________________________________.

ĐÁP ÁN ĐỀ THI ĐỀ XUẤT

I. 1 điểm : 0,2/1 câu trả lời đúng.


1. C 2. D 3. A 4. A 5. B

II. 2 điểm: 0,2/1 câu trả lời đúng.

1. A 2. B 3. D 4. A 5. C

6. B 7. C 8. A 9. D 10. A

III. 2 điểm: 0,2/1 câu trả lời đúng.

1. have (you) been

2. playing

3. is going to rain

4. arrived

5. was still cooking

6. is held

7. climbed

8. is

9. is always complaining

10. to be admitted

IV. 1,5 điểm : 0,25/1 câu trả lời đúng.

1 C 2A 3D 4A 5C 6B

V. 1điểm : 0,2/1 câu trả lời đúng.

They start and finish their full time education from the age of five to sixteen.

Many parents send their children to state schools.

No, they don’t.

At eleven every child had to sit for the eleven –plus examination.

The children with the highest marks at the eleven –plus examination.



VI. 1,5 điểm : 0,3/1 câu trả lời đúng.


1. A letter was sent to my friend in London last week.

2. I haven’t seen him since I was a student.

3. She reminded me to bring my passports with me when I went abroad.

4. Would you mind not smoking in the ward?

5. Because he was lazy, he failed the exam.

VII. 1 điểm : 0,2/1 câu trả lời đúng.

1. Instead of reusing plastic bags, we should / can use cloth bags.

2. Will you have a look at the house for me while I am on holiday?

3. We hope to give more green color to the town and earn some money for our group.

4. “Friends of the Earth” is an organization to protect our planet.

5. It was brave of him to jump into the river to save the child.



ĐỀ 15

I.Circle the word whose underlined part is pronounced differently from the others.( 1 pt
)

1. A. looked B. watched C. stopped D. carried

2. A. bath B. father C. theater D. health

3. A. study B. success C. surprise D. sugar

4. A. children B. charity C. charm D. champagne

5. A. danger B. angry C. language D. passage

II. Choose the word or phrase that best fits the blank space in each sentence. ( 2pts)

1. Someone….........tickets are free.

A. said me B. said me that C. told me D. told to me

2. When she lived in the countryside, she ……………….in the river.

A. was swimming B. used to swim C. has swum D. is used to swimming

3. Saigon ……………more than three hundred years ago.

A. was built B. has been built C. was building D. built

4. It is nearly ten years ……………….I saw her.

A. when B. since C. for D. while

5. Would you mind if I……………a photo ?

A. take B. took C. would take D. am going to take

6. Last week I ……………my children to the biggest zoo in town.

A. got B. brought C. fetch D. took

7. They said to me about the people and the places..........they saw in New York.

A. Which B. Who C. that D. whom

8. Are you proud…………….your country and its tradition ?

A. about B. on C. of D. for

9. Do you collect stamps or other things ?- Yes, I am a stamp…………………

A. collecting B. collector C. collect D. collection

10. I wish I...........a new computer.

A. had B. had had C. have D. would have

III. Give the correct tense or form of the verbs in brackets.( 2pts)

1. Mary spent hours (1. repair ) ...................................her bike.

2. They advised him ( 2.not/ cross )...................... the street aganist the red lights.

3. He and his sister (3.not/ meet) ........................my friends since last Christmas

4. She (4.work) ..........................................here since she left school ?

5. Everyone (5. read) this novel, let us ( 6.discuss) ....................................... it.

6. I (7. know) ..............her for six years, when I met her, she (8.work) ...................in a Law office.

7. They’d rather (9.walk) ...........................than (10.go) ..........................by car.

IV. Choose the word or phrase that best fits the blank space in the passage. ( 1,5pts)

For many people, traveling by plane is an exciting experience. Others, however, find the whole idea quite terrifying, (1) ................. flying is no more dangerous (2) ......................any other form of travel and some experts say it is considerably safer. It is known, however, that most accident occurs during taking off and landing when a (4) ......................decisions are vitally important.

The people (5) ............. job it is to look after the passengers, the stewards and stewardnesses play an important part in helping passengers to (7)................ safe and comfortable. Indeed for many passengers being taken such care of is all part of the total experience. No other form of travel involves waiting for people in quite the same (10) ................ , with food, drink, newspapers, magazines, music, and even video films.

1.A. althoughB. tooC. andD. because
2.A. asB. thanC. thenD. with
3.A. leader’sB. chief’sC. driver’sD. pilot’s
4.A. whoseB. whichC. theirD. that
5.A.experienceB. restC. feelD. lie
6.A. wayB. kindC. sortD. part
V. Read the passage and choose the best answer for the questions.(1pt)

Every year people in many countries learn English. Some of them are young children. Others are teenagers. Many are adults. Some learn at school. Others study by themselves. A few learn English just by hearing the language in films, on television, in the office or among their friends. Most people must work hard to learn English.

Why do all these people learn English? It’s not difficult to answer this question. Many boys and girls learn English at school because it is one of their subjects. Many adults learn English because it is useful for their work. Teenagers often learn English for their higher studies because some of their books are in English at the college or university. Other people learn English because they want to read newspapers or magazines in English.

1. According to the writer. English is learnt by...............

A. young children B. adults C. teenagers D. all are correct

2. In the line 2 of the second part, the word “it” refers to..........

A.country B. young children C.English D. question

3. Where do many boys and girls learn English?

A. at home B. at school C. in evening classes D. in the office

4. Why do adults learn English?

A. Because they want to see movies in English B. Because they need it for their job.

C. Because they are forced to learn it D. Because it’s not difficult to learn.

5. What of the following is NOT mentioned in the passage?

A. Children like reading English newspapers B. People in many countries learn English

C. English is one subject in school. D. Some books are written in English.

VI. Rewrite each sentence, beginning as shown, so that the meaning stays the same.(1,5pts)

1. They will have to change the date of the meeting again.

àThe date...............................................................................................................................

2. “I am going to meet my father in front of the station”, said Huy.

àHuy said that……………………………………………..…………………..……

3. Her mother can cook better than her.

àShe can’t………………………………………….……………………….…….

4. I don’t have a computer.

àI wish……………………………………..…….……………….…………………

5. I haven’t met him for two years.

àIt’s ........................................................................................................................................

VII. Use the words or phrases given to complete the sentences.(1pt)

1.There / used / be / Church / near / post office / my town.

...................................................................................................................................................................

2.While / I / open / letter /, phone / ring

...................................................................................................................................................................

3.I / be / very pleased / see / Tom / again / after / long time.

...................................................................................................................................................................4.You / look / thinner . You / lose / weight ?

...................................................................................................................................................................

5.That / my old house / I / born / grew up .

..................................................................................................................................................................

Key

I. 1 điểm : 0,2/1 câu trả lời đúng.

1. D 2. B 3. D 4. D 5. A

II. 2 điểm: 0,2/1 câu trả lời đúng.

1. C2. B3. A4. A5. B
6. D7. C8. C9. B10. A
III. 2 điểm: 0,2/1 câu trả lời đúng.

1. repairing 2. not to cross 3. haven’t met

4. Has she worked.......5. reads 6. dicuss 7. have known

8. worked 9. walk 10. go

IV. 1,5 điểm : 0,25/1 câu trả lời đúng.

1.A 2.B 3.D 4.A 5. C 6. A

V. 1điểm : 0,2/1 câu trả lời đúng.

1.D 2.C 3.B 4.B 5.A

VI. 1,5 điểm : 0,3/1 câu trả lời đúng.

1.The date of the meeting will have to be changed again.

2.Huy said that he was going to meet his father in front of the station.

3.She can’t cook as well as her mother.

4.I wish I had a computer.

5.It’s two years since I last met him.

VII. 1 điểm : 0,2/1 câu trả lời đúng.

1. There used to be a Church near the post office in my town

2. While I was opening the letter, the phone rang

3. I am very pleased to see Tom again after a long time.

4. You look thinner . Have you losen weight ?

5.That is my old house where I was born and grew up.



ĐỀ 16

I
/(1,0p)Choose the word in each group that has the underlined part pronounced differently from the rest.

1. A. nothing B. cover C. morning D. done

2. A. how B. town C. slow D. power

3. A. change B. chemistry C. choose D. cheese

4. A. match B. catch C. math D. watch

5. A. bread B. great C. break D. steak

II/ (3,0 points) Choose the best word or phrase to complete the following sentences.

1. I like reading books ______ tell about different peoples and their cultures.

A. who B. whom C. which D. where

2. “Buy me a dictionary on your way back,______?”

A. will you B. don’t you C. can’t you D. are you

3. Mary usually walks to school but today she ______ a bike.

A. ride B. rides C. riding D. is riding

4. She worked hard ______she passed the exam.

A. so B. if C. because D. as

5. I’ll give you a lift to the station if you _______ in a hurry.

A. were B. are C. will be D. would be

6. I’m feeling pretty tired. Do you mind ______ me home?

A. taking B. took C. take D. to take

7. Nam looks much _______ today than yesterday.

A. good B. well C. better D. the best

8. Scientists and engineers have invented devices to remove ______ from industrial wastes.

A. polluted B. pollute C. polluting D. pollutants

9. The dress you bought is of very good quality. It ______ very expensive.

A. may be B. must be C. should have been D. must have been

10. The Pikes put ______ their trip because of bad weather.

A. away B. up C. out D. off

11. Neither Linh nor her classmates ______ the National Museum so far.

A. visit B. visited C. have visited D. has visited

12. Nga: “You look nervous! ______” – Hoa: “This thunder scares me to death.”

A. How are you? B. Why’s that? C. Come on! D. What’s wrong?

13. He was homesick and he ______ all his family and friends.

A. missed B. wished C. desired D. hoped

14. I'm very ______ to you for putting in so much hard work.

A. thoughtful B. grateful C. considerate D. careful

15. If you come to the theatre late, you have to wait until the ______ to get in.

A. break B. refreshment C. interval D. half-time

III. Choose the underlined part (marked A, B, C or D) that needs correction.

21. I look forward to have the resolution to the problem I have mentioned.

A B C D

22. They have moved nothing in your room while they sent you to the hospital.

A B C D

23. Aren’t you afraid that they will sack you if you didn’t start coming to work on time?

A B C D

24. I had to drive to the factory to pick up my brother, who’s car wouldn’t start.

A B C D

25. It was not easy for us getting tickets for the concert.

A B C D

IV. Give the correct tense or form of the verbs in brackets.

26. When my mother came home from work, I_________(cook) dinner.

27. Jane_________(go) to school by bicycle every day.

28. The children_________(play) badminton in the stadium now.

29. You_________(go) to London last month?

30. The candidates mustn’t_________(bring) books into the examination room.

V/ (1,5 points) Read the passage and choose the best option A, B, C or D to answer the question.

Along with jogging and swimming, cycling is one of the best all-round forms of exercise. It can help to increase your strength and energy, giving you more efficient muscles and a stronger heart. But increasing your strength is not the only advantage of cycling. Because you’re not carrying the weight of your body on your feet, it’s a good form of exercise for people with painful feet or backs. However, with all forms of exercise it’s important to start slowly and build up gently. Doing too much too quickly can damage muscles that aren’t used to working. If you have any doubts about taking up cycling for health reasons, talk to your doctor and ask for his/her advice.

Ideally you should be cycling at least two or three times a week. For the exercise to be doing you good, you should get a little out of breath. Don’t worry that if you begin to lose your breath, it could be dangerous and there must be something wrong with your heart. This is simply not true; shortness of breath shows that the exercise is having the right effect. However, if you find you are in pain then you should stop and take a rest.

1. People with back problems might go cycling because__________.

A. it enables them to carry the weight of their body on their feet C. it helps to make their backs become stronger

B. it does not make them carry the weight of their body on their feet D. it helps them to relieve their backache

2. All forms of exercise must be started__________.

A. gradually B. quickly C. strenuously D. violently

3. According to the writer, it is best to go cycling__________.

A. once a week B. at least two or three times a week

C. at least two or three times a day D. every day

4. You should not worry about the shortness of breath because__________.

A. it shows that there is something wrong with your heart C. it is a sign of exercise having the right effect

B. it shows that you should stop and take a rest D. it is a sign of your getting rid of your heart problem

5. Which of the following is NOT included in the advantages of cycling?

A. Giving you a stronger heart B. Increasing you strength and energy

C. Giving you better muscles D. Making you look younger

VI/ (1,5 points) Fill in each blank with one suitable word.

Energy is one of the problems that many people are interested in. It is not an unfamiliar word. It is heard, said, discussed day after day. It is close to everyone’s (1)_____ life. You turn on the lamp and it is (2)______ that gives you light. You turn on a TV and it is energy that gives you pictures and sound. You ride a motorcycle and it is energy that gives you movement. You (3)______ your meals and it is energy that gives you heat to boil rice.

The problem is that the demand for energy is rising and that the price of energy is getting (4)______ and higher. The supply of energy on earth is limited. It cannot provide us all forever. The shortage of energy in the future is inevitable. Therefore, (5)______ energy is a must if we want to continue to live in a safe and sound world.

VII/ (1,5 points) Complete the following sentences, using the words or phrases suggested.

1. I / feel / hopeful / that / we / find / suitable house / very soon.

...................................................................................................................................................................

2. She / not / play / piano / well / as / her sister.

...................................................................................................................................................................

3. I wish / could / give you / all the expensive things / life.

...................................................................................................................................................................

4. You / hear / Maria / since / you / leave school?

...................................................................................................................................................................

5. The people / live / next door to us/ keep / have / all night parties.

...................................................................................................................................................................

VIII/ (1,5 points) Finish each of the following sentences the same as the sentence printed before it.

1. People are going to build a new library in the area.

à A new library _______________________________________________.

2. “Why don’t you take a day off tomorrow?” she said to me.

à She suggested that ____________________________________________.

3. The weather is bad so we can’t go camping.

à If the weather ________________________________________________.

4. He was punished because he behaved badly.

à Because of __________________________________________________.

5. The keeper had no sooner opened the case door than the lion attacked him.

àHardly ______________________________________________________.

Key

I
/ (1,0 điểm) Mỗi đáp án chọn đúng được 0,2 điểm.

* Lưu ý: Thí sinh chỉ cần viết đáp án A, B, C hoặc D

1. C. morning2. C. slow3. B. chemistry4. D. watch5. A. bread


II/
(3,0 điểm) Mỗi đáp án chọn đúng được 0,2 điểm.

* Lưu ý: Thí sinh chỉ cần viết đáp án A, B, C hoặc D

1. C. which2. A. will you3. D. is riding4. A. so5. B. are
6. A. taking7. C. better8. D. pollutants9. B. must be10. D. off
11. C. have visited12. D. What’s wrong?13. A. missed14. B. grateful15. C. interval
III. Choose the underlined part (marked A, B, C or D) that needs correction.

21. I look forward to have the resolution to the problem I have mentioned. having

A B C D

22. They have moved nothing in your room while they sent you to the hospital. since

A B C D

23. Aren’t you afraid that they will sack you if you didn’t start coming to work on time? don’t

A B C D

24. I had to drive to the factory to pick up my brother, who’s car wouldn’t start. whose

A B C D

25. It was not easy for us getting tickets for the concert. to get

VI. Give the correct tense or form of the verbs in brackets.

26. When my mother came home from work, I_________(cook) dinner. was cooking

27. Jane_________(go) to school by bicycle every day. goes

28. The children_________(play) badminton in the stadium now. are playing

29. You_________(go) to London last month? Did you go

30. The candidates mustn’t_________(bring) books into the examination room. bring

V/ (1,5 điểm) Mỗi đáp án chọn đúng được 0,3 điểm.

1. B2. A3. B4. C5. D
VI/ (1,5 điểm) Mỗi đáp án đúng được 0,3 điểm.

1. daily2. energy3. cook/ prepare4. higher5. saving
VII/ (1,5 điểm) Mỗi câu viết đúng được 0,3 điểm.

1. I feel hopeful that // we will/(can) find // a suitable house very soon.

2. She doesn’t/(can’t) play // the piano // as well as her sister.

3. I wish I // could give you all the expensive things // in life.

4. Have you // heard from Maria // since you left school?

5. The people who/(that) // live next door to us // keep (on) having all night parties.

The people living // next door to us // keep (on) having all night parties.

* Trong từng câu viết, nếu thí sinh có sai sót phần nào, Giám khảo trừ điểm thành phần của phần đó.

VIII/
(1,5 điểm) Mỗi câu viết đúng được 0,3 điểm.

1. A new library is // going to be // built in the area.

2. She suggested that I // (should) take a day off // the following day/(the next day).

3. If the weather weren’t // bad, we // could go camping.

If the weather were // fine/(nice/ better), we // could go camping.

4. Because of his bad // behavior //, he was punished.

Because of behaving // badly, // he was punished.

5. Hardly had // the keeper opened the case door when // the lion attacked him.

ĐỀ 17

A.PHONETICS. (1pt)I. Pick out the word whose underlined part is pronounced differently (0,6pt)

1. A. tradition B. pollution C. question D. action

2. A. invited B. lived C. learned D. played

3. A. mechanic B. knitting C. kitchen D. kettle

II. Choose the word that has different stress from others.(0,4pt)

1. A. tropical B. logical C. casual D.commercial

2. A. access B. control C.wander D. income

B.VOCABULARY AND STRUCTURE (4.5 pts)

I. Choose the correct answer A,B,C, or D to complete the sentences(2,0pts)


1. ‘Let’s go to Ha Long Bay on the weekend” “……………………….”

A. That’s a fine day B. That’s a good idea C. Yes, please D. Yes, let’s

2. People have talked a lot…… …..UFOs ………..many years.

A. from /in B. . in / for C. about/ for D. for/ for

3.” I wish I………your village again some day” Lisa told Mary

A. could visit B. can visit C. visited D. will visit

4. She is the …………..student in his class.

A. good B. the well C. the best D. best

5. …………….we’ve got a few minutes to wait for the train, let’s have a cup of coffee.

A. Since B. A & C are correct C.As D. A & C are wrong

6. People in Israel are going to celebrate their festival,……..is called Passover.

A. whose B.who C. which D. where

7. ………..I was really tired , I couldn’t sleep.

A. Even though B. So C. Therefore D. Because of

8. A funnel-shaped storm passing overland below a thunderstorm is called a……...

A. typhoon B. tsunami C. tornado D. hurricane

9. You have read this article on the website,…………?

A haven’t you B don’t you C aren’t you D didn’t you

10 Thuy’s grandmother wants her to……………the volume on TV.

A: switch on B:turn on C:turn up D:turn down.

II. Choose the underlined word or phrase that needs correcting.(1p)


1.His good sense of humorous distinguishes him from his brother.

A. his B. humorous C. distinguishes D. from

2. Between 1980 to 1990, the area was hit by the five disastrous tornado.

A. to B. the C. was hit D. disastrous

3.The secretary whom sits at the first desk on the right can give you the information.

A. whom B.sits C.at D.on

4. Who will look for your little sister when your mother goes out?

A. who B. look for C.when D.goes

5. If she get up early, she will go to school on time

A. get B. late C.will go D.on

II.Put the verbs into correct tense or form(1,5 p)

1/His family has bought a lot of food. They(go)………..……..a trip to Hue next week.

2. I (phone)…….. …..…...you as soon as I arrive in Ho Chi Minh City.

3.Jane really enjoys(invite)……………….to dinner parties.

4. When I came, he (listen)……………..……..…….to music.

5.I (write)…….…..to my pen pal two months ago., but I (not receive )……….. ....his reply since then.

C. READING (2,5pt)

I. Fill each of the numbered blanks in the following passage. Use only one word in each space
.(1pt)

was problem children example It
Population growth is a serious (1)...........around the world. At the beginning of the 20th century, there were about 1.5 billion people in the world. In 1984 the world population (2)..........4.8 billion people. By the year 2000, .(3)…… will be about 6.1 billion. This growth in population is not happening everywhere For (4)……. in Europe the population is not growing at all families in these countries are smaller now. Only about 2.1 (5)...........are born for every woman.

II.Read the following passage and answer the following questions. (1,5pt)

Solar energy is a long lasting source of energy which can be used almost anywhere. To generate solar energy, we only need solar cells and the sun! Solar cells can easily be installed on house roofs, so no new space is needed and each user can quietly generate their own energy. Compared to other renewable sources, they also possess many advantages. Wind and water power rely on turbines which are noisy, expensive and easy to break down. Solar cells are totally silent and non-polluting. As they have no moving parts, they require little maintenance and have a long lifetime.

However, solar energy also has some disadvantages. We can only generate solar energy during daytime because the system depends on sunlight. Besides, solar cells require large area to work effectively. The main disadvantage of solar energy is that it costs about twice as much as traditional sources such as coal, oil, and gas. This is because solar cells are expensive. Scientists are hoping that the costs of solar cells will reduce as more and more people see the advantages of this environmentally friendly source of energy.

1. What does the passage primarily discuss?

A. Advantages and disadvantages of solar energy.B. Solar energy’s advantages over other sources of energy.

C. The cost of solar energy. D. Solar energy as an alternative for fossil fuels.

2. What does the word they in line 4 refer to?

A. solar energy users. B. other renewable resources. C. advantages. D. solar cells.

3. What is NOT mentioned as an advantage of solar cells?

A. They require little maintenance. B. They are non-polluting.

C. They cost little to produce. D. They operate quietly.

4. Which of the following is NOT a traditional source of energy?

A. solar energy B. oil C. coal D. gas

5. Which of the following is the main disadvantage of solar energy mentioned in the passage?

A. It is expensive. B. Solar cells require large areas to operate.

C. It is unfriendly to the environment. D. It depends on sunlight.

6.What does the word advantages in line 12 mean ?

A. very bad. B. resources C. benefits D. friendly

D. WRITING (2,0 pt) I.Rewrite the following sentences .(1,0 pt)

1. France presented the Statue of Liberty to the USA in 1876.

The Statue of Liberty …………………………………………….

His car can't run as fast as mine.

=> My car ……………………………………………….

3. The deer doesn’t eat meat and hippopotamus doesn’t eat meat either.

Neither…………………………………………………………….

4.The girl was so beautiful that everyone admired her.

=> It was such ............................................................

5.“Do you live here?”Bill asked

=> ………………………………………………………….

II. Complete sentences, using the given words /phrases(1 p)

1./ Mai has/ be/ able /sing/.since/she/ was/twelve.

………………………………………………………………

2/ I recently went back / the town where /I /born..

……………………………………………………………..

3/ He/ used / go fishing /when he/ small.

………………………………………………………

4/I/study/harder/If/ I/ be/ you.

=> ……………………………………………………

5.Why/ not/we/go/the cinema ?

=> ……………………………………………………………

Key

A.PHONETICS. (1pt)

I. Pick out the word whose underlined part is pronounced differently from that of the other words (0,6pt)


1. C. question

2. A. invited

3. B. knitting

II. Choose the word that has different stress from others.(0,4pt)

1. D.commercial

2. B. control

B.VOCABULARY AND STRUCTURE (4.5 pts)

I. Choose the correct answer A,B,C, or D to complete the sentences(2,0pts)


1. D. Yes, let’s

2. C. about/ for

3.A. could visit

4. D. best

5. B. A & C are correct

6. C. which

7. A. Even though

8. C. tornado

9. A haven’t you

10. C:turn up

II. Choose the underlined word or phrase that needs correcting.(1p)

1.B. 2. A. 3.A. 4. B. 5. A.

II.Put the verbs into correct tense or form(1,5 p)

1/ are going .

2. will phone

3.being invited

4. was listening

5. wrote

6. haven’t received

C. READING (2,5pt)

I. Fill each of the numbered blanks in the following passage. Use only one word in each space
.(1pt)

1. problem

2. was

3.It

4. example

5. children

II.Read the following passage and answer the following questions. (1,5pt)

1. A.
Advantages and disadvantages of solar energy.

2. D. solar cells.

3. C. They cost little to produce.

4. A. solar energy

5. A. It is expensive.

6. C. benefits

D. WRITING (2,0 pt)

I.Rewrite the following sentences which have the same meaning with the given, using the suggested words at the beginning.(1,0 pt)


1. The Statue of Liberty was presented to the USA in 1876 by France

2. My car can run faster than his/his car.

3. Neither the deer not hippopotamus eat meat

4. It was such a.beautiful girl .that everyone admired her...

5. Bill asked me if /whether I live there.

II. Complete sentences, using the given words /phrases(1 p)

1. Mai has been able to sing since she was twelve

2. I recently went back to the town where I was born..

3. He used to go fishing when he was small .

4. I would study harder if I were you.

5. Why don't we go to the cinema ?



ĐỀ 18

I. Choose the word whose underlined part is pronounced differently from the others.

1. A. claimed B. warned C. occurred D. existed

2. A. health B. appear C. ready D. heavy

3. A. tidal B. sight C. mineral D. describe

4. A. though B. throw C. through D. thought

5. A. thunder B. erupt C. trust D. pull

II. Choose the best answer from the four options given to complete each sentence.

6. I can’t understand the French visitors. I wish I________French.

A. knew B. will know C. know D. have known

7. The church________about 100 yearsAgo.

A. is built B. was built C. will be built D. has been built

8. People in Israel are going to celebrate their festival________is called Passover.

A. whose B.who C. which D. where

9. ________I was really tired , I couldn’t sleep.

A. Even though B. So C. Therefore D. Because of

10. Lan is very tired. ________, she has to finish herAssignment before going to bed.

A. Although B. So C. Therefore D. However

11. I suggest________some money for poor children.

A. raise B. to raise C. raised D. raising

12. She asked me if I________a laptop computer the following day.

A.buy B. will buy C. bought D. would buy

13. I ________telephone her if I knew her number.

A. would B. have to C. will D. shall

14. Honda motorbikes________in Viet Nam.

A. produce B. will produce

C. are produced D. would be produced

15. We have learnt English________2001.

A. for B. since C. in D. during

16. We________already________Huong Pagoda.

A. were / seeing B. Have / seen

C. are / seeing D. Will / see

17. All the houses in the area________immediately.

A. has to rebuilt B. had to rebuild

C. have to be rebuilt D. have to rebuild

18. If he________soon, he might miss the train.

A. isn’t coming B. doesn’t come

C. won’t come D. didn’t come

19. Mr. Long said that he________in Ho Chi Minh City.

A. lived B. is living C. has lived D. will live

20. Your sister works in a foreign company, ________she?

A. isn’t B. didn’t C. wasn’t D. doesn’t

III. Identify the underlined word/ phrase (A or B,C,D) that needs correcting to become an exact one.

21. My sister enjoys read about wild animals and natural mysteries.

A B C D

22. Mr. Thach who sing English songs very well is my teacher of English.

A B C D

23. My father asked us not to spending too much time playing computer games.

A B C D

24. Ba can play the piano better more than his friends can.

A B C D

25. We can save nature resources by using solar energy.

A B C D

IV. Choose the correct answer from the options below, then complete the following passage,

Today, supermarkets are found in almost every large city in the world. But the first supermarket (26)_________opened only fifty years ago. It was opened in New York by a man named Michael Cullen. A supermarket is different (27)___________other types of stores in several ways. In supermarkets, goods are placed on open shelves. The(28)__________ choose what they want and take them to the checkout counter. This means that fewer shop assistants are needed than in other stores. The way products are displayed is another difference between supermarkets and many other types of stores; (29)__________ example, in supermarkets, there is usually a display of small inexpensive items just in front of the checkout counter: candies, chocolates, magazines, cheap foods and so on. Most customers (30)__________go to a supermarket buy goods from a shopping list. They know exactly what they need to buy. They do the shopping according to a plan.

26. A. is B. has been C. was D. were

27. A. in B. from C. of D. with

28. A. customers B. managers C. assistants D. sellers

29. A. in B. for C. of D. by

30. A. who B. what C. which D. whom

V. Complete the second sentence so that it has a similar meaning to the first one.

31. They have just sold that old house.

=> That old house .................................................................................................................

32. In spite of the bad weather, they had a wonderful holiday.

=> Although ..........................................................................................................................

33. They will build a new mall here.

=> A new mall ......................................................................................................................

34. Unless he takes these pills, he won’t be better.

=> If ......................................................................................................................................

35. Despite working hard, he can’t support his large family.

=> Although ..........................................................................................................................

VI. Complete each of the following sentences, using the words given.

36. If/ it/ not rain/ tomorrow/ I/ go/ camping/ my friends.

=> ..........................................................................................................................................

37. If/ I/ meet/ alien/ outer space/ I/ invite/ home/ talk.

=> ..........................................................................................................................................

38. I/ never/ travelled/ by/air.

=> ..........................................................................................................................................

39. The students/ study/ for exam/ now.

=> ..........................................................................................................................................

40. It/ be/ such/ cold day/ we/ decide/ not/ go out.

=> ..........................................................................................................................................

ĐÁP ÁN

1. D 2. B 3. C 4. A 5. D 6. A 7. B 8. C 9. A 10. D

11. D 12. D 13. A 14. C 15. B 16. B 17. C 18. B 19. A 20. D

21. B read => reading 22. B sing => sings 23. B spending => spend

24. C more than => than 25. B nature => natural

26. C 27. B 28. A 29. B 30. A

31. That old house has just been sold (by them).

32. Although the weather was bad, they had a wonderful holiday.

33. A new mall will be built here (by them).

34. If he does not take these pills, he won’t be better.

35. Although he works hard, he can’t support his large family.

36. If it does not rain tomorrow, I will go camping with my friends.

37. If I metAnAlien from outer space, I would invite him/her/it to my home and talk/ to talk.

38. I have never travelled by air.

39. The students are studying for their exam now.

40. It was such a cold day that we decided not to go out.

ĐỀ 19

I. Choose one word whose underlined part is pronounced differently from the others (0,5m) .


1. A. enjoyed B. collected C. visited D. needed

2. A. house B. youth C. outside D. sound

3. A. passed B. watched C. played D. washed

4.A.ocean B.cotton C.chopstick D.solid

5.A.songs B.mountains C.plants D.samples

II. Choose the best answer (2ms).

1. This machine _______ used since last week.

A. has beenB. isC. wasD. will be
2. If he _______ hard, he will pass his math test next week.

A. studyB. studiesC. studiedD. is studying
3. My brother got wet ________ he didn’t bring the umbrella with him.

A. butB. soC. becauseD. and
4. They say that solar energy doesn’t cause _________.

A. pollutionB. polluteC. pollutedD. pollutant
5. Peter fell over while he _______ basketball.

A. is playingB. was playingC. playedD. plays
6.The people _______ live in England speak English.

A. whomB. whichC. whoD. whose
7. Be _______! He is looking at you.

A. carefullyB. careC. carefulD. carelessly
8. Jane speaks English _______ than you do.

A. more fluentlyB. more fluentC. more fasterD. more better
9. Let’s go out for a walk, _________ ?

A. don’t weB. do weC. shall weD. will we
10. Nam: “Congratulations on your success!” Hoa: “__________.”

A. You’re welcomeB. No, thanksC. That’s very kind of youD. Yes, of course
III. Complete the sentences by writing the correct tense of the verbs in blankets(1,0m).

1. My brother ___________________ (buy) this house 10 years ago.

2. My friend always _________________ (get) up at 5 a.m.

3. Marry _________________ (clean) the room when I arrived yesterday.

4. John _________________ (use) this motorbike since 2002.

5. If he ______________________ (not come) soon, we will miss the train.

IV. Find the word(s) (marked A, B, C or D) that is incorrect in each of the following sentences(0,5m)

1. Although she doesn’t trust weather forecasts, but she likes watching them.

A B C D

2. I suggested to help elderly people with their chores.

A B C D

3. She has worked in this company since I am a child.

A B C D

4. Mrs. Phuong, whom sings very well, is my English teacher.

A B C D

5. My brother went to Japan for two months ago.

A B C D

V. Write the correct form of the word in each blank.(1.0)

1. We often take part in many ___________________ activities at school. (culture)

2. The village fair is open for _________________. (entertain)

3. Their _______________ has lasted a lifetime. (friend)

4. She is poor but she lives in _________________. (happy)

5.My children are very _________________. about their summer vacation. (excite)

VI. Complete the following passage by choosing A, B, C or D to fill in each blank(1.0).

Almost a hundred thousand people were killed (1) ________half a million homes were destroyed as a result of an earthquake in Tokyo in 1923. The earthquake began a minute before noon (2) _______ people were cooking their midday meals. Thousands of stoves (3) _______ overturned as soon as the earth began to shake. As a result, small fires broke out everywhere and quickly spread. It was impossible (4) _______fire fighting equipment. Consequently, over ninety percent (5) _______ the damage was caused by fire rather than by the collapse of buildings.

1. A. but B. and C. as D. so

2. A. when B. what C. where D. which

3. A. are B. was C. will be D. were

4. A. using B. use C. to use D. used

5. A. of B. in C. with D. to

VII. Read the passage and then answer the questions below(2.0).

Mark Twain was a famous American writer. His real name was Samuel Langhorne Clemens and “Mark Twain” was his pen name. He was born in a small town on the Missouri River in the USA. The boy had many friends at school and when he became a writer, he described them in his stories.

When he was twelve, his father died and the boy began to work and learned the profession of a printer. He always wanted to be a sailor and when he was twenty, he found work on a river boat. Then he left the boat and lived in California. Here he began writing short stories under the name of Mark Twain. He sent them to newspapers. The readers liked his stories very much. His best novel “The Adventure of Tom Sawyer” was published in 1876.

1. Who was Mark Twain?

…………………………………………………………………………

2. Where was he born?

………………………………………………………………………………

3. How old was he when his father died?

……………………………………………………………………………………

4. Did the readers like his stories very much?

……………………………………………………………………………

5. When was his best novel published?

………………………………………………………………………………………………………

VIII. Finish each of the following sentences in such a way that it means exactly the same as the sentence printed before it. (2.0)

1. “Let’s go out for dinner” Mary said.

à Mary suggested ____________________________________________.

2. John is too young to understand the question.

à John is not ________________________________________________.

3. The last time I heard from my sister was 5 months ago.

à I haven’t _________________________________________________.

4. The box was so heavy that my son couldn’t carry it.

à It was ____________________________________________________

5. Columbus discovered America.

àThe explorer…………………………………………………….………..



KEY to 19

I. 1.A 2. B 3.C 4.C 5.C

II.1.A 2.B 3.C 4.A 5.B 6.C 7.C 8.A 9.C 10.C

III.1. bought 2.gets 3. was cleaning 4.has been used 5.doesn’t come

IV.1. C 2. A 3. C 4. A 5. C

V.1. cultural 2. entertaining 3. friendship 4. happiness 5. excited

VI.1. B. and 2. A. when 3. D. were 4. C. to use 5. A. of

VII.1. Mark Twain( He ) was a famous American writer.

2. He was born in a small town on the Missouri River in the USA.

3. He was twelve.

4. Yes, they did.

5. It / His best novel was published in 1876.

VIII.1. Mary suggested going out for dinner.

John is not old enough to understand the question.

I haven’t heard from my sister for 5 months .

4.It was such a heavy box that my son couldn’t carry it.

5. The explorer who discovered America is Columbus .



ĐỀ 20

I/. Choose the word that has the underlined part pronounced differently
.

1. A. campus B. publish C. supply D. difficult

2. A. about B. south C. young D. count

II/. Choose the word that its main stress is placed differently from the others.

3. A. model B. admire C. happy D. cover

4. A. scenery B. hamburger C. pagoda D. grocery

III/. Choose the word (A, B,C or D) that best completes the sentence. (2,5)

It’s very kind ……..you to help me.

A. of B. to C. with D. for

A free …….. is guaranteed to every citizen.

A. educated B. education C. educating D. educational

AIDS is a newly-discovered and very …………………disease.

A. danger B. dangerous C. endangered D. dangerously

Please ………your cigarette. I’m allergic to smoke.

A. put aside B. put off C. put up D. put out

Let’s have a drink,………..?

A. shall we B. will we C. shall you D. do we

They didn’t go on a picnic ……………the weather was awful.

A. so B. although C. because D. because of

Do you know the lady………..son is standing over there?.

A. who B. whom C. which D. whose

I believe you because I know you are ………...

A. true B. truth C. truthful D. truly

Water ……… at 100 degrees Celsius.

A. boiling B. boil C. is boiling D. boils

Laziness is normally the cause of ……….

A. poor B. poverty C. richness D. success

IV/. Choose the words or phrases that are not correct in standard English:

When she came to my house this morning, I still slept.

A B C D

Is Fiona used to work late at the office?

A B C D

He was punished because he did the test careless yesterday.

A B C D

The doctor told my father stop smoking because of his sickness..

A B C D

V/ Find the once choice that best completes the passage below :

Rice is (19)……….. by Vietnamese people every day. It often (20)………. in tropical countries such as Vietnam, Thailand or Malaysia. The Chinese have also been growing rice for (21)…………. years. The seeds are planted in special beds to grow into young rice plants. Then they are taken to fields covered (22)…………. muddy water called paddies. The fields of rice look very (23)…………….. After 3 or 5 months, the rice is ready to be picked. People often drain away water before colleting rice. Eating rice is a special action in the world. They don’t use spoons or forks to enjoy bowls of rice. (24)………… , they use two short sticks known as chopsticks to put rice into their mouths. China and Vietnam are the four countries in which people use chopsticks very well.

A. used B. taken C. eaten D. boiled

A. grows B. keeps C. plants D. stays

A. thousands B. thousand C. thousand of D. thousands of

A. in B. by C. with D. of

A. beauty B. beautiful C. beautifully D. the beauty

A. However B. Moreover C. Besides D. Instead

VI/ Read the following passage and choose the best answers.

Children’s education is changing rapidly today. In the past, teachers made children sit still for hours. They made them memorize all sorts of things. In other words, children had to go on repeating things until they knew them by heart. Today, many teachers wonder if it possible to make children learn at all. They say you can only help them learn. They say you must let children learn and discover things for themselves.

What did teachers make children do in the past?

A. stand for hours B. memorize everything C. repeat their homework D. sit for days

Children in the past were mase to learn everything…………………

A. by head B. by hand C. by hair D. by heart

Nowadays, many teachers say that they only ……………………

A. give children more homework B. make children learn C. help children D. teach children at home

Today, the modern learning method is ……………………

Letting children play computer games B. making children read a lot of books

C. doing homework for children D. letting children discover things for themselves

VII/ Complete the second sentences without changing the meaning of the first sentences.

No one in the group is younger than Mai..

Mai is……………………………………..

“Where are you going for your holidays?” I asked them.

I asked them………………………………………………………………………………….

The luggage is too heavy for her to carry.

The luggage is so…………………………………………………………………….

He is tired, so he couldn’t score any goals.

If he…………………………………………………………………………………………….

I think it would be good idea to take the train.

I suggested …………………………….

“Why don’t you go with me?” he asked me.

He asked ……………………………………………………………………………….

Lan often stayed up late when she was young.

Lan used ………………………………….

Does it take you 3 hours to do this task?

Do you ………………………………………………………………………………….

I can’t buy this bike because I don’t have enough money.

If I have ………………………………………………………………………………….

I can’t play basketball well because of my height.

Because I ……………………………………………………………………………….

Learning English is interesting.

It ……………………………………………….

No one has answered the questions yet.

The question …………………………………….

III/. Choose the word (A, B,C or D) that best completes the following passage.

Environmental (41)……………is one of the most serious problems (42)……………mankind today. Air, water and soil are necessary to the survival of all (43)………………things. Badly polluted air can (44)………………illness and even death. Polluted water (45)…………… many kinds of wild animals and other marine life. Pollution of soil reduces the amount of land (46) ……………growing food.

41.A. pollute B. polluted C. pollution D. polluting

42.A. face B. faces C. to face D. facing

43.A. lived B. living C. live D. lives

44.A. cause B. give C. make D. catch

45.A. kills B. kill C. are killing D. doesn’t kill

46.A. of B. to C. from D. for

* Answer the questions.

47.What is the most important problem of mankind nowadays?

…………….…………………………………………………………………………………….

48.Which kinds of pollution are mentioned in this passage?

……………..…………………………………………………………………………………….

49.Why do many kinds of wild animals and other marine life die?

………………………………………………………………………………………………….

50. Does the pollution of soil make the amount of food-growing land narrower?

…………….……………………………………………………………………………

ĐỀ 21

I/. Choose the word that its main stress is placed differently from the others.


1. A. energy B. appliance C. pesticide D. minimize

2. A. cover B. pollute C. reduce D. provide

3. A. prevent B. install C. suggest D. solar

4. A. pollution B. dynamite C. production D. protection

5. A. change B. charity C. chopstick D. character

6. A. depend B. poet C. equal D. region

7. A. think B. thick C. this D. thanks

8. A. shoulder B. should C. could D. would

II/. Choose the word that its main stress is placed differently from the others.

9. A. pagoda B. grocery C. institute D. benefit

10. A. control B. remote C. access D. advance

III. Choose the correct word or phrase in each of the following sentences

11. This newspaper is ………every day. It’s a daily newspaper.

A. publishing B. published C. publish D. to publish

12. We are taking about the preservation of ………resources.

A. natural B. nature C. naturally D. naturalize

13. Everyone must take part in ………forests and increase forestation.

A. protect B. protecting C. protection D. protected

14. In order to save electricity, an ordinary 100-watt light bulb can be replaced by ________ .

A. an energy-saving bulb B. a 1000-watt light

C. an electric bulb D. a saving - energy bulb

15. ………we know her address, we will call you.

A. Unless B. Since C. If D. Therefore

16. If you know where she lives, let me………….

A. to know B. knowing C. know D. knew

17. We’ll make the beach clean and ………… again

A. polluted B. dirty C. awful D. beautiful

18. It is impossible …………a newspaper without reading about the damage we are doing to the environment.

A. open B. opening C. to open D. opened

19. If you are late again, you…………the job.

A. won’t lost B. will lose C. lost D. will be lost

20. She never goes to the movies…………she is very busy.

A. so B. because C. if D. because

21.Do you often go out …………Saturday evenings?

A. in B. at C. on D. for

22 .Hoa likes reading and ……………

A. so is Minh B. Minh is too C. so does Minh D. Minh does either

23 .They don’t like milk and ………………

A. neither do we B. so do we C. we do too D. neither don’t we

24. I have two brothers and we are ……………at school.

A. both B. all C. either D. neither

25. The man………………we saw yesterday is a scientist.

A. whom B. whom C. which D. whose

26. I feel ……………in this book.

A. interesting B. interested C. interest D. interestingly

27.Vietnam is a …………country.

A. develop B. developed C. developing D. development

28.There is …………time. Let’s hurry.

A. a lot of B. few C. a few D. not much

29.This house is ………………of the three.

A. old B. older C. oldest D. the oldest

30.If it …………, we’ll have the party outside.

A. rains B. doesn’t rain C. rained D. didn’t rain

IV/ Find the once choice that best completes the passage below :

Among the festivals (31)………… by some of Asian people is the Moon Cake Festival, also known (32)………… the Mid August Festival. Large numbers of small round moon cakes are eaten (33)…………. this day, and children enjoy carrying colorful (34)……………. lanterns come in all shapes; the most popular ones are shaped like fish, rabbits and butterflies. According to (35)…………, the moon shines the brightest on the night of the Moon Cake Festival. As the moon rises, tables are placed (36)……………… the house and women make offerings of fruit and moon cakes to the Moon Goddess.

31. A. celebrated B. made C. held D. set

32.A. like B. as C. such as D. Þ

33.A. in B. for C. at D. on

34.A. wood B. metal C. paper D. gold

35.A. they B. them C. it D. their

36.A. under B. near C. outside D. around

V/. Choose the words or phrases that are not correct in standard English:

37. Did the house broken when you were away?

A B C D

38.That is the girl whom Jim wants to marry her.

A B C D

39. Although it rained heavy, they went out last night.

A B C D

40. The accident happened in front of my house last night because the driver’s carelessness.

A B C D

VI/ Read the following passage and choose the best answers.

I often hear or read about “natural disaster”- the eruption of Mount St Helen, a volcano in the state of Washington: Hurricane Andrew in Florida; the floods in the American Midwest; terrible earthquakes all over the world; huge fires; and so on. But I’ll never forget my first personal experience with the strangeness of nature – “the London Killer Fog” of 1952. It began on Thursday, December 4th when a high –pressure system (warm air) cover southern England. With the freezing-cold air below, heavy fog formed. Pollution from factories, cars, and coal stoves mixed with the fog. The humidity was terribly high, there was no breeze at all. Traffic (cars, trains, and boats) stopped. People couldn’t see, and some walked onto the railroad tracks or into the river. It was hard to breathe, and many people got sick. Finally on Tuesday, December 9th , the wind came and the fog went away. But after that, even more people got sick, many of them died.

41.Which natural disaster isn’t mentioned in the text?

A. a volcanic eruption B. a flood C. a hurricane D. a tornado

42.What is his unforgettable person experience?

A. the London killer B. the heavy fog in London C. the strangeness of nature D. a high-pressure system

43.What didn’t happen during the time of the “London Killer Fog”?

A. pollution B.humidity C.heavy rain D.heavy fog

44.The traffic stopped because of ………….

A.The rain B. the windy weather C. the humid weather D. the heavy fog

VII/ Complete the second sentences without changing the meaning of the first sentences.

45. Hurry up or you will be late

If……………………………………………………………………………….

46. Helen can play the piano better than Elizabeth.

Elizabeth…………………………………………………………………………………………….

47. Peter failed the exam because he was lazy.

Because of………………………………………………………………………………………..

48. Please don’t play your music loudly.

Would you mind………………………………………………………………………………….

49. They didn’t have a map, so they got lost.

Because………………………………………………………………………………………….

50. “Do you know my teacher’s telephone number, Lan? Said Minh.

Minh asked…………………………………………………………………………………….



ĐỀ 22

I/. Choose the word that has the underlined part pronounced differently
.

1. a. push b. pull c. rush d. butcher

2. a. route b. shout c. trousers d. amount

II/. Choose the word that its main stress is placed differently from the others.

3. A. tutor B. highlight C. lunar D. enroll

4. A. selection B. national C. recycle D. convenient

III/. Choose the word (A, B,C or D) that best completes the sentence.

The boys looks very proud …………his success at school.

A. of B. at C. about D. on

Is Dalat rather crowded?- No, there are …………people than in HCM.

A. few B. fewer C. more D. less

In spite of ………late, he arrived in time.

A. he started B. his being started C. he was started D. starting

It’s time the children ………here now.

A. are B. are being C. were D. to be

I won’t go …………you explain everything to me.

A. unless B. although C. because D. if

How many languages are there ……………the world?

A. through B. over C. for D. in

Your son is not only intelligent ……………… handsome.

A. but also B. and C. but D. with

The washing machine was very expensive, ………we couldn’t afford to buy it.

A. though B. because C. therefore D. so

This is …………novel I’ve read.

A. bad B. good C. better D. the worst

They don’t understand the matter ; ……………they didn’t ask for help.

A. moreover B. therefore C. however D. but

IV/ Find the once choice that best completes the passage below :

Nowadays, people are destroying rain forests of the earth seriously. It is (15)……………….. that every year 100,000 (16)……………….. kilometers of rain forests are destroyed for (17)……………….. of wood paper and fuel as well as for the residence and (18)……………….. land. Rain forests are very important for the world’s climate. They receive the rainfall on the earth and produce a large amount of the world’s oxygen. Destroying rain forests,(19)……………….. , is destroying our environment. Saving rain forests is a(n) (20)……………….. problem. Nations need cooperation to save rain forests, if not, it will be late.

A. exhausted B. pleasure C. interesting D. estimated

A. square B. cross C. round D. heart

A. bring B. supply C. support D. suggest

A. planting B. field C. farming D. rice

A. moreover B. however C. so D. therefore

A. national B. international C. world wide D. world

V/. Choose the words or phrases that are not correct in standard English:

When I was a boy, I was used to go fishing with my father.

A B C D

My sister, Lan can neither sing or swim.

A B C D

She always wears modern and fashioned clothes when she is at work

A B C D

The last time he saw in public, he worn a great suit.

A B C D

VI/ Read the following passage and choose the best answers.

The zipper is a wonderful invention. They are very common so we forget that they are wonderful. They are strong, but they open and close very easily. They come in many colors and sizes. In 1810s, people in the US wore high shoes or clothes with long row of buttons. It was hard for them to wear anything. They wanted an easier way to put on and take off clothes. White comb Judson invented the first zipper in 1839. He called it a slide fastener.

A zipper has three parts. There are dozens of metal or plastic teeth in two rows. These are fastened to two flexible strips of cloth. A fastener slides along and fastens the teeth together. When it slides the other way, it takes the teeth apart.

Many people forget that zippers are wonderful because ………………………

A. they are strong B. they open and close easily C. they are colorful D. they are common

The first zipper was invented ………………………………………

A. in 1800 B. in the 1800s C. in 1839 D. in 18th century

A zipper consists of …………………………………

A.three metal of plastic teeth B. dozens of parts C. dozens of parts D. metal or plastic teeth, two flexible strips of cloth and fastener

It was …………for people in the US to wear clothes or shoes with a long row of buttons.

A.easy B. difficult c.wonderful d.convenient

VII/ Complete the second sentences without changing the meaning of the first sentences.

29. The weather is too terrible for you to go out.

If the weather …………………………………….……………………………………………..

30. All the students have to take the final exam.

The final exam………..………………………………….

Please don’t repeat what I said.

Would you mind…… … …………………………………… ….

I can’t swim as well as my friend can

My friend……………………………………………………….

33. We lost our way. We didn’t arrive on time.

Unless………………………………………………………………………………………………

It took me three hours to open the door.

We spend…………………………………………………

I can’t answer all the questions.

I wish……………………………………………………

36. The film was too boring for you to watch

The film was so……………………………………………………………………………

They don’t play football any more.

They used…………………………………………………………

38. The man said to me,“Please tell me the way to the nearest post office?’

The man asked me………………………………………………………………………………

All of the buildings in this town aren’t as high as yours.

Your building………………………………

The show was interesting to the boy.

The boy was……………………………………………………

41. My father doesn’t smoke any more.

My father used……………………………………………………

V/. Choose the words or phrases that are not correct in standard English:

42. Your sister is different with you, isn’t she?

A B C D

43. His family was very poor, because he had to work for a company.

A B C D

44. You know where Lan is , do you ?

A B C D

45. When you want to go fishing this morning , I’ll go with you.

A B C D

46. I’ve watched the film what is about the life on other planets

A B C D

47. If I met an alien, I would invited him to my home and talk

A B C D

48. Minh asked me how far is it from the airport to my house.

A B C D

49. If I were three inches taller, I would apply on that job.

A B C D

50. How about to use public buses instead of cars.

A B C D



ĐỀ 23

I - Choose the word whose underlined part is pronounced differently from the others.


1. A. passed B. watched C. played D. washed

2. A. proud B. about C. around D. would

3. A. market B. depart C. card D. scare

* Choose the word which has a different stress pattern

4. a. award b. prevent c. visit d. except

5. a. bookshelf b. advanced c. above d. depend

II – Choose the best answer from the four options given (A, B,C, or D) to complete each sentence.

III- Choose the right answer (1.6 p)


6. A: Congratulations! You did great. B:_________________

A. It's nice of you to say so. B. It's my pleasure.

C. You're welcome. D. That's okay.

7. “ Do you have a bike ?” “ No, but I wish I____________ one.”

A. having B. have C. can have D. had

8. I spent half a year________________ this boat.

A. to build. B. building C. built D. on building

9. If I____________ any problem, I______________ ask for your help.

A. has / will B. had / will C. have / would D. have / will

10. People in Israel are going to celebrate their festival________is called Passover.

A. whose B.who C. which D. where

11. Lan is very tired. ________, she has to finish her assignment before going to bed.

A. Although B. So C. Therefore D. However

12. We have learnt English________2001.

A. for B. since C. in D. during

13. Your sister works in a foreign company, ________she?

A.isn’t B. didn’t C.doesn’t D. wasn’t

14. Hoai can not remember the name of the restaurant_________she ate her favorite roasted duck.

A. which B. whose C. whom D. where

15. If I were a flower, I_______a sunflower.

A. was B. were C. will be D. would be

V- Complete these sentences

1. If / I / rich, / I / travel / around / world / family. => ………………………………………………………….

2. Mr John / never / allow / daughter / swim / river / her friends

=> ……………………………………………

I / told / not / be late / class / next time.

=> ……………………………………………………..……………

Yuri Gagarin / be / first man / who / travel / space.

=> …………………………………………………….…

It / so / hot / yesterday / that / we / not / sleep.

=> ……………………………………………………………

IV. Give the correct form of verbs given.

A. -John ....................... (21. lose) his job last month and since then he ............................. (22. be) out of work.

- Do you know why he ……………………….... (23. lose) his job?

- Because he ................................. (24. be) very rude to his boss.

B. Yesterday morning, when I .......................... (25. arrive) at the airport, Sophie .............................. (26. wait) for me.

She ................................ (27. wear) a pink dress and ......................................... (28. look) very pretty.

V - Write the correct form of the word in the parentheses.

29.Many people became……………….…….after the earthquake. (home)

30.There are many…………………………throughout the year. (celebrate)

31.The scientists can predict the ............................... of a volcano (erupt)

32. We stayed at home because it rained ………………………….. (heavy)

33.I am looking forward to ……………………………………..from you. (hear)

VI - Read the following passage, then choose the correct answer to questions 26 - 30.

I went to Australia on a student program last year and I like to (34)_____you about it. I was very (35)_____when I knew I was going to Australia because I had never been there before. I didn’t think about the problems of speaking English (36)_____I met my host family. At first I couldn’t communicate with them because my English was so bad. All the five years I had been learning English wasn’t much used at all (37)_____we didn’t have real practice at school. Even though my grammar was good, my pronunciation wasn’t. My problem is (38)_____ ‘l’ and ‘r’. For example, Australian people often asked “What do you eat in Vietnam?” I wanted to tell them that we eat rice, but they didn’t understand when I said “We eat lice”…

34 .A. say B. tell C. talk D. speak

35. A. exciting B. excites C. excited D. excite

36. A.after B. until C. when D. while

37.A. although B. even C. because D. so

38 A. pronouncing B. speaking C. reading D. telling

VII- Complete the second sentence so that it has a similar meaning to the first one.

39 People say that they bought this shop last year.

=> It is___________________________________________.

- They are_____________________________________________________.

"How much do you think it will cost?" he said to me.

=> He asked me____________________________________.

Mr.Brown's team has lost the game.He looks very sad.

=> Mr. Brown whose_______________________________.

42.Do you know the man who sat next to me at Nam’s birthday party last night?

- You know____________________________________________________.

It takes Minh 2 hours to do his homework every day.

=> Minh spends______________________________________.

44 .We were late for school because of the heavy rain.

=> Because it_________________________________________.

My mother used to us clean the house.

=> We used_____________________________________________________.

46."I'm working in a restaurant, and don't care much for it."she said

- She said_______________________________________________________.

47. It’s two years since I last spoke to her.

=> I haven’t__________________________________________________

Going swimming in the river in the summer is interesting.

=> It_______________________________________.

HƯỚNG DẪN CHẤM ĐỀ THI ĐỀ XUẤT

I - (1 điểm) : 0,2 điểm / ý

Choose the word whose underlined part is pronounced differently from the others.

C. played 2. D. Would 3. D. Scare

4. A. entrance 5. B. Love

II - (1,5 điểm) : 0,1 điểm / ý

Choose the best answer from the four options given (marked A, B,C, or D) to complete each sentence.

6. C. was built 7. A. advised 8. A. who

9. A. a seven-room 10. D. won't be 11. C. used to go

13. C. not to spend 14. D. Where 15. D. Would be

III- (1 điểm) : 0,2 điểm / ý

Identify the underlined word/ phrase (A or B,C,D) need correcting to become an exact one.

16. The picture was painting by Michael last year. (was painted)

A B C D

17. There’s the woman who she sold me the handbag. (NOT she)

A B C D

18. Mr. Smith is going to buy a new Japanese car, doesn’t he? ( isn’t he )

A B C D

19. I met a lot of interesting people while I was studying at Ho Chi Minh City. ( in )

A B C D

20. If I were you, I didn’t buy that expensive car. ( wouldn’t )

A B C D

IV. (2 điểm) : 0,25 điểm / ý

Give the correct form of verbs given.

21. lost22. has been23. lost24. was
25. arrived26. was waiting27. was wearing28. looked
V (1 điểm) : 0,2 điểm / ý - Write the correct form of the word in the parentheses.

29.Many people became…………after the earthquake. (homeless)

30.There are many………………throughout the year. (celebrations)

31.The scientists can predict the ................. of a volcano (eruption)

32. We stayed at home because it rained …………….. (heavily)

33.I am looking forward to ………………………..from you. (hearing)

VI (1 điểm) : 0,2 điểm / ý

- Read the following passage, then choose the correct answer to questions 26 - 30.

34 B. tell 35. A. exciting

36. B. until 37. C. because 38. A. pronouncing

VII (2,5 điểm) : 0,25 điểm / ý

I. Complete the second sentence so that it has a similar meaning to the first one.

39. - It is said that they bought this shop last year.

Cách 2-They are said to have bought this shop last year.

40. - He asked me how much I thought it would cost.

41. - Mr. Brown whose team has lost the game looks very sad.

42. - You know the man who sat next to me at Nam’s birthday party last night, don’t you?

43 - Minh spends 2 hours doing his homework every day.

44. - Because it rained heavily, we were late for school.

45. - We used to be made to clean the house by my mother.

46. - She said that she was working in a restaurant and didn’t care much for it.

47. - I haven’t spoken to her for two years.

48. - It is interesting to go swimming in the river in the summer.

ĐỀ 24

Choose the word (A, B, C, D) whose underlined part is pronounced differently from the others. (0.5 point)


Question 1: A. invited B. needed C. ended D. liked

Question 2: A. climate B. ethnic C. unit D. city

Choose the word (A, B, C, D) whose main stress is placed differently from that of the others. (0.5 point)

Question 3: A. likely B. lovely C. kiddy D. apply

Question 4 :A. instruct B. decide C. contain D. common

Choose the word/phrase (A, B, C, D) that best fits the space in each sentence. (2.5 points)

Question 5: Look! The boys …………………… basketball in the school yard.

A. play B. are playing C. played D. were playing

Question 6: A runny nose, sneezing and coughing are the …………………… of common cold.

A. materials B. measures C. medicines D. symptoms

Question 7: The toy …………………… my father bought for my brother is very expensive.

A. who B. whom C. which D. whose

Question 8: The president is going to pay a state visit to Japan, ……………… he?

A. does B. doesn't C. is D. isn't

Question 9: Remember …………………… the instruction carefully before you use it.

A. reading B. to read C. read D. for reading

Question 10: She has worked as a secretary …………………… she graduated from college.

A. before B. since C. when D. until
Question 11: The boy's family is very poor. He has to go to school on foot. He wishes he……………………enough money to buy a bike.

A. has B. will have C. had D. has had

Question 12: Did the Second World War last from 1939 …………………… 1945?

A. in B. at C. on D. to

Question 13: It is very noisy. I can't hear what he is saying. Can you …………………… the radio?

A. turn up B. turn off C. turn on D. turn around
Question 14: Mai and Lan are attending a party.

- Mai: "Would you like some more Coke?" - Lan: " ………………… .”

A. I think so B. I'm not sure C. Yes, let's D. Yes, please

Choose the underlined part (A, B, C, D) that needs correcting. (1.0 point)

Question 15: I have to go to the dentist's because of I have a toothache.

A B C D

Question 16: Alex Ferguson, that is the most successful coach in Manchester United's history, underwent an emergency operation last month.

A B C D
Question 17: The Caspian Sea, a salt lake, is the largest than any other lakes in the world.

A B C D

Question 18: The woman said to her son that he can go out when he finished all his homework.

A B C D

Give the correct form of the word given in each sentence. (1.0 point)

Question 19: Lam will try to have a big …………………… of stamps. (collect)

Question 20: This bus is used for …………………… students to school. (take)

Question 21: An …………………… film will help you feel less depressed. (interest)

Question 22: Ha Noi and areas to the north will be …………………… tomorrow. (sun)

Read the following passage and mark the letter A, B, C, D on your answer sheet to indicate the correct word or phrase that best fits each of the numbered blanks. (1.0 point)

The Mekong river


The Mekong River, (23) ………………… Southeast Asia, is the world's 12th—longest river and the 7th longest in Asia.

It's about 4.350 kilometres (24) …………………… and flows through six countries, including China, Myanmar, Laos, Thailand, Cambodia and Vietnam. When flowing into Vietnam, the Mekong River is also called Cuu Long River,

(25) …………………… has two branches: Tien River and Hau River. People build houses and run their businesses on the rivers. The Mekong River is (26)……………… to more than 850 kinds of freshwater fish.

Question; 23 A. in B. on C. by D. at

Question 24: A. long B. length C. lengthen D. longest

Question 25: A. that B. whom C. which D. where

Question 26: A. home B. place C. region D. country

Read the following passage and choose the letter A, B, C, D to indicate the correct answer to each of the questions. (1.0 point)

The Americans are keen to win the race to send human beings to Mars. In 1992, the new boss of NASA, Dan Goldin, called on the American people to be the first to send explorers to another planet in the solar system. He reminded them of the symbolic gift carried to the moon and back by the Apollo 11 mission. It bears a message intended for the crew of the first spaceship to visit Mars. Goldin thinks it is time to begin the preparation for this historic journey. His speech echoed the words of the President, who promised that in 2019, 50 years after Neil Armstrong became the first man to set foot on the moon, the first astronaut would stand on Mars.

By the end of the twentieth century, various unmanned spaceships will have thoroughly investigated the surface of the planet. But, however clever a robot may be, it cannot match the type of information which can be gained -from direct human experience. The first geologist on the moon, Harrison Schmitt, was capable of interpreting the story of the landscape on the spot. Until humans walk on the red deserts of Mars, we will not be able to determine the history of this frozen world in any detail.

Question 27: Who called on the Americans to be the first to send explorers to another planet in the solar system?

The president

Dan Goldin

Neil Armstrong

D. Harrison Schmitt
Question 28: According to the American President, when would the first astronaut probably stand on Mars?

A. 1969

B. 1992

C. 2019

D. 2050

Question 29: According to the passage, by the end of the twentieth century, many …………… will have thoroughly investigated the surface of the planet.

manned spaceships

astronauts

robots

D. unmanned spaceships
Question 30: According to the passage, which of the following statements is NOT true?

A. A clever robot and a human being can provide the same information from Mars.

B. The first geologist on the moon was Harrison Schmitt.

C. We will not be able to determine the history of Mars in any detail until humans walk on it.

D. The Americans are keen to win the race to send human beings to Mars.

Rearrange the word(s) in a correct order to make complete sentences. (1.0 point)

Question 31: to you / I / for a long time. / have not written

Question 32: will take place/ from 14 June to 15 July 2018./ The 2018 FIFA World Cup/ in Russia

Complete the sentence so that it has a similar meaning to the original one. (1.5 points)

Question 33: He gave my sister a smart phone on her birthday.

=> My sister was __________________________
Question 34: Watching "Lat mat: Ba chang khuyet" is very exciting.

=> It is __________________________________
Question 35: It isn't nice, so we can't go for a picnic.

=> If it _________________________________

ĐỀ 25

I. PHONETICS

(1-3). Choose the word whose underlined part pronounced differently from that of the others.


1.a. meaningb. reasonc. featured. pleasant
2.a. champagneb. choicec. exchanged. children
3.a. denyb. studyc. typicald. pretty
(4-5). Choose the word whose main stress pattern is not the same as that of the others.

4.a. earthquakeb. energyc. extensived. damage
5.a. sometimesb. reductionc. studentd. interesting
II. GRAMMAR AND VOCABULARY

(6-20). Choose the word or phrase (a, b, c or d) that best fits the blank space in each sentence.


6. The final examination will be held __________ July 10th,2008.

a. inb. onc. atd. to
7. The children are excited ________ their upcoming trip to the zoo.

a. tob. aboutc. ford. with
8. Sixteen people showed ______ for the volleyball training session.

a. onb. upc. overd. through
9. she was an hour late, she didn’t apologize.

a. In spite ofb. Even thoughc. Howeverd. Because
10. They haven’t ______ an AIDS vaccine.

a. yet developedb. developed yet
c. developed alreadyd. already developed
11. People do exercise ______ fit.

a. to keepb. keepingc. keptd. in keeping
12. A person that you make friends with by writing letter is called a __________.

a. co-operatorb. cousinc. close friendd. penpal
13. You look so depressed. You look ________ you didn’t have a friend in the world.

a. as ifb. if onlyc. even ifd. although
14. Most parents think chatting on the Internet is _______

a. time-releaseb. time-wastingc. time-savingd. time-consuming
15. It really annoys me when people forget_________ thank you.

a. to sayb. sayingc. saidd. to saying
16. Don’t stay up late,______ ?

a. do youb. won’t youc. will youd. shouldn’t you
17. If I were in charge, I_____ things differently.

a. had doneb. will doc. would dod. would have done
18. Would you mind if I a friend to the party?

a. bringb. will bringc. broughtd. would bring
19. My father stopped________two years ago.

a. smokeb. to smokec. smokingd. smoked
20. ‘You got the first prize. Congratulations!’ ‘_________ ’

a. You’re welcomeb. Never mindc. It’s my pleasured. Thanks a lot


(21-25). Choose the underlined word or phrase (A, B, C or D) that needs correcting.

21. Could (A) you phone me (B) as soon (C) as he’ll come (D) back?

22. Would you (A) please stop to make (B) so (C) much noise (D)?

23. Last summer I have staved (A) on (B) my uncle (C) farm for (D) two weeks.

24. What (A) difficult (B) to master a (C) foreign language (D)!

25. The police said (A) that they had (B) reacted as fastly (C) as they could (D).

(26-30). Use the correct form of the word given in each sentence.

26. If you need any help, you can ask Tom. He’s very _____________ . (help)

27. I have a _____________ to talk a lot whenever I am nervous, (tend)

28. Gas and oil _____________ always increases in cold weather, (consume)

29. We find advertising on television very _____________ (effect)

30. The students wear their school uniforms with _____________. (proud)

III. READING

(31-38). Choose the word or phrase (a, b, c or d) that best fits the blank space in the following passage.


Most of the energy we use today (31)_______ from coal, oil and gas. But these will not last for ever, and burning them is slowly harming the (32) We need to (33)_______ other ways of supplying energy. Solar Power is a way of using the (34)_______ energy as heat or to make electricity. We can also use wind-power by building modern (35)_______ that spin in the wind. There are several types of water-power: river water in mountainous areas can (36)_______ to generate hydroelectric power, and we can also create electricity (37)_______ sea water flowing in and out with the (38)_______

31. a. makesb. createsc. comesd. begins
32. a. soilb. atmospherec. waterd. resources
33. a. look afterb. look intoc. look atd. look for
34. a. sun’sb. moon’sc. star’sd. earth’s
35. a. windbreaksb. windmillsc. wind tunnelsd. wind chimes
36. a. useb. usingc. be usedd. to use
37. a. inb. forc. byd. from
38. a. crestsb. waysc. wavesd. tides


(39 – 43). Read the passage below and then decide whether the statements that follow are True or False.

Many people now think that teachers give students too much homework. They say that it is unnecessary for children to work at home in their free time. Moreover, they argue that most teachers do not properly plan the homework tasks they give to students. The result is that students have to repeat tasks which they have already done at school.

Most people agree that homework is unfair. A student who can do his homework in a quiet and comfortable room is in a much better position than a student who does his homework in a small, noisy room with the television on. Some parents help their children with their homework. Other parents take no interest at all in their children’s homework.

It is important, however, that teachers talk to parents about homework. A teacher should suggest suitable tasks for parents to do with their children. Parents are often better at teaching their own children!

39. Many parents would like their children to have less homework.

40. Parents think that students should do a lot of work in their leisure time at home.

41. A lot of homework has not been planned properly, according to many

42. Only a small number of people think that homework is fair.

43. Teachers suggest parents should teach their own children at home.

IV. WRITING

(44 – 46). Choose the sentence (a, b, c or d) that is almost the same in meaning as the sentence given.

44. ‘I’m leaving here for Hue tomorrow,’ Hoa said.

a. Hoa said that she is leaving there for Hue the day after.

b. Hoa said that she was leaving there for Hue the day after.

c. Hoa said that she would leave there for Hue the day aftey.

d. Hoa said that she would leave here for Hue the day after.

45. It’s no use reading that book.

a. You should read that book.

b. That book has not been used.

c. You shouldn’t use that book for reading.

d. That book is not worth reading.

46. I don’t really want to spend my vacations in France.

a. I would rather not spend my vacations in France.

b. I would like to spend my vacations in France.

c. I don’t often spend my vacations in France.

d. I prefer spending my vacations in France.

(47-50). Rewrite the sentences so that they are nearest in meaning to the sentence printed before them.

47. He will only phone if he changes his mind.

He won’t ____________________________________________________

48. I haven’t been to Bristol for three years.

The last time _________________________________________________

49. ‘Please sit down,’ the teacher said to his students.

The teacher _________________________________________________

50. He is too short to play basketball.

He’s so _____________________________________________________

Đáp án

I.


1. d2. a3. a4. c5. b
II.

6. b7. b8. b9. b10. a
11. a12. d13. a14. d15. a
16. c17. c18. c19. c20. d
21. D (comes)22. B (making)23. A (stayed)24. A (how)25. C (fast)


26. helpful27. tendency28. consumption29. effective30. pride
III.

31. c32. b33. d34. a35. b
36. c37. d38. d
39. T40. F41. T42. T43. F
IV.

44. b45. d46. a
47. He won’t phone unless he change his mind

48. The last time I went to Bristol was three years ago./ The last time I was in Bristol was three year ago.

49. The teacher asked/ told his students to sit down

50. He’s so short that can’t play basketball.

ĐỀ 26

I. PHONETICS

(1-3). Choose the word whose underlined part pronounced differently from that of the others.


1.a. meaningb. reasonc. featured. pleasant
2.a. champagneb. choicec. exchanged. children
3.a. denyb. studyc. typicald. pretty
(4-5). Choose the word whose main stress pattern is not the same as that of the others.

4.a. earthquakeb. energyc. extensived. damage
5.a. sometimesb. reductionc. studentd. interesting
II. GRAMMAR AND VOCABULARY

(6-20). Choose the word or phrase (a, b, c or d) that best fits the blank space in each sentence.


6. The final examination will be held __________ July 10th,2008.

a. inb. onc. atd. to
7. The children are excited ________ their upcoming trip to the zoo.

a. tob. aboutc. ford. with
8. Sixteen people showed ______ for the volleyball training session.

a. onb. upc. overd. through
9. she was an hour late, she didn’t apologize.

a. In spite ofb. Even thoughc. Howeverd. Because
10. They haven’t ______ an AIDS vaccine.

a. yet developedb. developed yet
c. developed alreadyd. already developed
11. People do exercise ______ fit.

a. to keepb. keepingc. keptd. in keeping
12. A person that you make friends with by writing letter is called a __________.

a. co-operatorb. cousinc. close friendd. penpal
13. You look so depressed. You look ________ you didn’t have a friend in the world.

a. as ifb. if onlyc. even ifd. although
14. Most parents think chatting on the Internet is _______

a. time-releaseb. time-wastingc. time-savingd. time-consuming
15. It really annoys me when people forget_________ thank you.

a. to sayb. sayingc. saidd. to saying
16. Don’t stay up late,______ ?

a. do youb. won’t youc. will youd. shouldn’t you
17. If I were in charge, I_____ things differently.

a. had doneb. will doc. would dod. would have done
18. Would you mind if I a friend to the party?

a. bringb. will bringc. broughtd. would bring
19. My father stopped________two years ago.

a. smokeb. to smokec. smokingd. smoked
20. ‘You got the first prize. Congratulations!’ ‘_________ ’

a. You’re welcomeb. Never mindc. It’s my pleasured. Thanks a lot
(21-25). Choose the underlined word or phrase (A, B, C or D) that needs correcting.

21. Could (A) you phone me (B) as soon (C) as he’ll come (D) back?

22. Would you (A) please stop to make (B) so (C) much noise (D)?

23. Last summer I have staved (A) on (B) my uncle (C) farm for (D) two weeks.

24. What (A) difficult (B) to master a (C) foreign language (D)!

25. The police said (A) that they had (B) reacted as fastly (C) as they could (D).

(26-30). Use the correct form of the word given in each sentence.

26. If you need any help, you can ask Tom. He’s very _____________ . (help)

27. I have a _____________ to talk a lot whenever I am nervous, (tend)

28. Gas and oil _____________ always increases in cold weather, (consume)

29. We find advertising on television very _____________ (effect)

30. The students wear their school uniforms with _____________. (proud)

III. READING

(31-38). Choose the word or phrase (a, b, c or d) that best fits the blank space in the following passage.


Most of the energy we use today (31)_______ from coal, oil and gas. But these will not last for ever, and burning them is slowly harming the (32) We need to (33)_______ other ways of supplying energy. Solar Power is a way of using the (34)_______ energy as heat or to make electricity. We can also use wind-power by building modern (35)_______ that spin in the wind. There are several types of water-power: river water in mountainous areas can (36)_______ to generate hydroelectric power, and we can also create electricity (37)_______ sea water flowing in and out with the (38)_______

31. a. makesb. createsc. comesd. begins
32. a. soilb. atmospherec. waterd. resources
33. a. look afterb. look intoc. look atd. look for
34. a. sun’sb. moon’sc. star’sd. earth’s
35. a. windbreaksb. windmillsc. wind tunnelsd. wind chimes
36. a. useb. usingc. be usedd. to use
37. a. inb. forc. byd. from
38. a. crestsb. waysc. wavesd. tides
(39 – 43). Read the passage below and then decide whether the statements that follow are True or False.

Many people now think that teachers give students too much homework. They say that it is unnecessary for children to work at home in their free time. Moreover, they argue that most teachers do not properly plan the homework tasks they give to students. The result is that students have to repeat tasks which they have already done at school.

Most people agree that homework is unfair. A student who can do his homework in a quiet and comfortable room is in a much better position than a student who does his homework in a small, noisy room with the television on. Some parents help their children with their homework. Other parents take no interest at all in their children’s homework.

It is important, however, that teachers talk to parents about homework. A teacher should suggest suitable tasks for parents to do with their children. Parents are often better at teaching their own children!

39. Many parents would like their children to have less homework.

40. Parents think that students should do a lot of work in their leisure time at home.

41. A lot of homework has not been planned properly, according to many

42. Only a small number of people think that homework is fair.

43. Teachers suggest parents should teach their own children at home.

IV. WRITING

(44 – 46). Choose the sentence (a, b, c or d) that is almost the same in meaning as the sentence given.

44. ‘I’m leaving here for Hue tomorrow,’ Hoa said.

a. Hoa said that she is leaving there for Hue the day after.

b. Hoa said that she was leaving there for Hue the day after.

c. Hoa said that she would leave there for Hue the day aftey.

d. Hoa said that she would leave here for Hue the day after.

45. It’s no use reading that book.

a. You should read that book.

b. That book has not been used.

c. You shouldn’t use that book for reading.

d. That book is not worth reading.

46. I don’t really want to spend my vacations in France.

a. I would rather not spend my vacations in France.

b. I would like to spend my vacations in France.

c. I don’t often spend my vacations in France.

d. I prefer spending my vacations in France.

(47-50). Rewrite the sentences so that they are nearest in meaning to the sentence printed before them.

47. He will only phone if he changes his mind.

He won’t ____________________________________________________

48. I haven’t been to Bristol for three years.

The last time _________________________________________________

49. ‘Please sit down,’ the teacher said to his students.

The teacher _________________________________________________

50. He is too short to play basketball.

He’s so _____________________________________________________

Đáp án

I.


1. d2. a3. a4. c5. b
II.

6. b7. b8. b9. b10. a
11. a12. d13. a14. d15. a
16. c17. c18. c19. c20. d
21. D (comes)22. B (making)23. A (stayed)24. A (how)25. C (fast)


26. helpful27. tendency28. consumption29. effective30. pride
III.

31. c32. b33. d34. a35. b
36. c37. d38. d
39. T40. F41. T42. T43. F
IV.

44. b45. d46. a
47. He won’t phone unless he change his mind

48. The last time I went to Bristol was three years ago./ The last time I was in Bristol was three year ago.

49. The teacher asked/ told his students to sit down

50. He’s so short that can’t play basketball.

ĐỀ 27

I. PHONETICS

(1-3). Choose the word whose underlined part pronounced differently from that of the others.


1.a. famousb. spacec. paced. attack
2.a. booksb. catsc. dogsd. maps


3.a. houseb. hourc. holed. humor
(4-5). Choose the word whose main stress pattern is not the same as that of the others.

4.a. linguisticsb. ordinaryc. teenagersd. graduating
5.a. relaxb. recognizec. realized. relatively
II. GRAMMAR AND VOCABULARY

(6-20). Choose the word or phrase (a, b, c or d) that best fits the blank space in each sentence.


6. Mike is always proud _____ his success at school.

a. onb. ofc. atd. in
7. Many companies participated________ the trade fair.

a. onb. ofc. ind. to
8. Never put______ till tomorrow what you can do today.

a. off

b. over

c. back

d. away

9. They went on playing_______ it started to rain.

a. though

b. because

c. but

d. despite

10. In the 18th century, workers loved wearing jean because it did not_______

a. break off

b. tear off

c. wear out

d. come out

11. We’ve got of time, so there’s no need to rush.

a. very much

b. a number

c. great deal

d. plenty

12. Shut the window, _____ it’ll get too cold in here.

a. unless

b. if not

c. otherwise

d. though

13. If she_____ rich, she would travel around the world.

a. would be

b. is

c. has been

d. were

14. The polluted river is smelly and filthy.

a. dangerous

b. shallow

c. dirty

d. swollen

15. I wish you_____making that noise. It’s bothering me.

a. would stop

b. will stop

c. stop

d. can stop

16. I expect_______ a postcard from my pen friend in England today.

a. to receive

b. receiving

c. to be received

d. being received

17. _____ you tell me how to get to the nearest supermarket?

a. May

b. Could

c. Do

d. Should

18. She is ______ to lift such a heavy bag.

a. not enough strong

b. enough strong

c. not strong enough

d. strong not enough

19. The equipment in our office needs________ .

a. to modernize

b. modernizing

c. modernized

d. modernization

20. Computers _____ to do a lot of jobs these days.

a. are used

b. used to

c. are using

d. use

(21-25). Choose the underlined word or phrase (A, B, C or D) that needs correcting.

21. If only (A) I would (B) play the guitar as well (C) as you (D).

22. Don’t (A) forget turning (B) off all the (C) lights before you go to (D)

23. My father wants to go (A) back to the places (B) where (C) he used to visit (D).

24. My family lived (A) in Ha Noi since (B) 1990 to 1998, but now (C) we are living (D) in Ho Chi Minh City.

25. Mary asked me if (A) I go (B) to school on foot (C) or by bike (D).

(26-30). Use the correct form of the word given in each sentence.

26. Energy-saving bulbs make____________ use of electricity, (efficiency)

27. The most __________ earthquake in Japanese history occurred in 1923. (disaster)

28. For many employees, job____________is more important than making

money, (satisfy)

29. People fled from the earthquake area in_____________ . (terrify)

30. Pompeii was completely destroyed in A.D.79 by an_____________of Mount

Vesuvius, (erupt)

III. READING

(31-38). Choose the word or phrase (a, b, c or d) that best fits the blank space in the following passage.


In the country of China, there is a wall that is 1,500 miles (31)_______ . It is called the Great Wall of China. It (32)_______ uphill and down, through valleys and mountains. Every inch of this 1,500-mile wall (33)_______ made by hand. The people of China made it to keep (34)________ their enemies. There are watch (35)_____ all along the way. The wall, is made of brick and earth. It is high and wide on top. People can walk along the top (36)________ it were a road. It is said that it (37)_______ ten years to build one part of this wall. No other defense line has ever been made as (38)_______ as the Great Wall of China.

31. a. longb. lengthc. lengthyd. lengthen
32. a. comesb. movesc. windsd. lasts
33. a. isb. arec. wasd. were
34. a. offb. outc. ind. up
35. a. buildingsb. housesc. boxesd. towers
36. a. asb. ifc. as ifd. even if
37. a. tookb. spentc. maded. lasted
.38. a. longb. longerc. longestd. length
(39-43). Read the passage below carefully and choose the correct answer a, b, c or d.

I get a lot of letters at this time of the year from people complaining that they have a cold which won’t go away. There are so many different stories about how to prevent or cure a cold that it’s often difficult to know what to do. Although colds are rarely dangerous, except for people who are already weak, such as the elderly or young babies, they are always uncomfortable and usually most unpleasant. Of course, you can buy lots of medicines which will help to make your cold less unpleasant, but you must remember that nothing can actually cure a cold or make it go away faster. Another thing is that any medicine which is strong enough to make you feel better could be dangerous if you are already taking drugs for some other illness so always with your chemist or doctor to see whether they are all right for you. And remember they might make you sleepy – please don’t try to drive if they do! Lastly, as far as avoiding colds is concerned, whatever you may be told about magic foods or drinks, the best answer is to keep strong and healthy – you’ll have less chance of catching a cold, and if you do, it shouldn’t be so bad!

39. This is from__________ .

a. doctor’s notebookb. a diary
c. a magazined. a school biology book
40. What is the writer’s intention?

a. to write in an amusing wayb. to give general advice
c. to complain about coldsd. to describe personal experience
41. Who should talk to the doctor before buying medicine for a cold?

a. People who are already weak.

b. People who catch a bad cold?

c. People who drive to work.

d. People who are already taking drugs.

42. What is the writer’s opinion of ‘magic food and drink’?

a. The writer believes in it.

b. The writer doesn’t believe in it.

c. The writer is concerned about it.

d. The writer is interested in it.

43. Which of the following is NOT true?

a. Colds are not very often dangerous.

b. Colds cannot be cured or prevented.

c. Colds are uncomfortable and unpleasant.

d. Colds might make you sleepy.

IV. WRITING

(44-46). Choose the sentence (a, b, c or d) that is almost the same in meaning as the sentence given.


44. He used to write home once a week.

a. He enjoys writing home every week.

b. He never fails to write a weekly letter home.

c. He doesn’t now write home once a week.

d. He was forced to write home every week.

45. Bridges is by far the richest man I know.

a. He is the richest man in my country.

b. He is one of many very rich men I know.

c. He is richer than all his friends.

d. He is much richer than anyone else I know.

46. Because of hard working, she feel ill.

a. She was too ill to work hard.

b. She did not work, so she fell ill.

c. She was not ill although she worked hard.

d. She worked so hard that she fell ill.

(47-50). Rewrite the sentences so that they are nearest in meaning to the sentence printed before them.

47. People use money for buying and selling goods.

Money_________________________________________________________

48. Tm sorry I broke the glass,’ Peter said to Jane.

Peter apologized_________________________________________________

49. He is intelligent, but he doesn’t do well at school.

He doesn’t do well at school________________________________________

50. If you run a lot, you will get fitter.

The more_______________________________________________________

Đáp án

I.

1. d2. c3. b4. a5. a
II.

6. b7. c8. a9. a10. c
11. d12. c13. d14. c15. a
16. a17. b18. c19. b20. a
III.

21. B (could)22. B (to turn)23. C (which/ that)
24. B ( from)25. B (went)
26. efficient27. disastrous28. satisfaction
29. terror30. eruption
III.

31. a32. c33. c34. b35. d
36. c37. a38. a


39. c40. b41. d42. b43. d
IV.

44. c45. d46. d
47. Money is used for buying and selling goods.

48. Peter apologized to Jane for breaking the glass.

49. He doesn’t do well at school though/ although/ even though he is intelligent.

50. The more you run, the fitter you get.

ĐỀ 28

(1-3). Choose the word whose underlined part pronounced differently from that of the others.


1. a. laughb. caughtc. naughtyd. taught
2. a. designb. solarc. websited. sample
3. a. receivedb. watchedc. discoveredd. destroyed
(4-5). Choose the word whose main stress pattern is not the same as that of the others.

4. a. permanentb. powerc. permissiond. carpet
5. a. photographyb. minorityc. heroicd. amateur
II. GRAMMAR AND VOCABULARY

(6-20). Choose the word or phrase (a, b, c or d) that best fits the blank space in each sentence.


6. The town of Gouda is famous_____its cheese.

a. onb. toc. fromd. for
7. He was happy to be_______ friends again.

a. amongb. inc. neard. off
8. Were you brought ______ in the city or in the country?

a. offb. forwardc. upd. over
9. The teacher made Jane ______ up and answer his question.

a. standb. standingc. to standd. stands
10. You’d better leave for the airport now_______ there’s a lot of traffic on the way.

a. in caseb. in orderc. in factd. in fact
11. There was so many things that we never get .

a. interesting – boringb. interested – bored
c. interested – boringd. interesting – bored
12. I turn on the radio_____listen to the news.

a. so thatb. in orderc. so asd. in order to
13. I can speak______ words of French, but I can’t write it.

a. littleb. a littlec. fewd. a few
14. If I had time, I______ to the countryside with you this weekend.

a. will gob. would goc. wentd. would have gone
15. My uncle_____ you met yesterday is an engineer.

a. whichb. whatc. whomd. whose
16. Don’t waste your breath________ with him.

a. arguingb. arguec. for arguingd. to argue
17. When we came to visit her last night, she___________

a. is watchingb. was watchingc. has watchedd. watched
18. You were not listening in class, ?

a. were youb. weren’t youc. was itd. wasn’t it
19. I’ll go to the town tomorrow, and_________ .

a. so will my sisterb. my sister will either
c. neither will my sisterd. will my sister too
20. I’ll pass me the newspaper? ~ Sure. Here you are.

a. Would you mindb. Could you pleasec. May youd. Why don’t you
(21-25). Choose the underlined word or phrase (A, B, C or D) that needs correcting.

21. I’ve been (A) looking forward to see (B) you again since (C) we last met (D)

22. Her novel, that (A) was published (B) last month, is (C) one of the best-sellers. (D)

23. We spent (A) an interested (B) holiday in (C) Ha Long Bay last summer (D).

24. Many (A) people have complain (B) about (C) the dirt from (D) the factory.

25. She refused (A) to tell (B) us (C) where was she (D) going.

(26-30). Use the correct form of the word given in each sentence.

26. The talk was both_________ and entertaining, (inform)

27. Our school is sending three _________ to the meeting, (represent)

28. This singer is not very pretty but she sings very_______. (beautiful)

29. Mel Gibson is a__________ actor, (talent)

30. There are significant ________ between America English and British English (differ)

III. READING

(31-38). Choose the word or phrase (a, b, c or d) that best fits the blank space in the following passage.


Online gaming site Roiworld (31)________ 600 teens ages 13 to 17 in late April and found that teens spend two hours per day online on average, 80% of which is spent using a (32)________ network. These same teens are, however, showing signs of “Facebook Fatigue.” Nearly one in five (19%) who have an account (33)_____ visit Facebook or are using it less. Of the group that are saying goodbye to Facebook, 45% have (34)______________ interest, 16% are leaving because their parents are there, 14% say there are “too many adults/older people” and 13% are concerned about the (35)____________ of their personal information. While interest in Facebook may be waning, it’s still the most popular social network (36)_____ teens – 78% have created a profile and 69% still use it. YouTube (37)____ second; 64% of teens claim to have a YouTube profile and continue to use the site. MySpace comes in a distant third (41%) and Twitter takes the fourth (38) (20%).

31.a. workedb. suggestedc. surveyedd. admited
32.a. commonb. supportivec. nationald. social
33.a. no longerb. any morec. once mored. any time
34.a. developedb. lostc. takend. pursued
35.a. firmnessb. publicc. sourced. privacy
36.a. withinb. betweenc. amongd. around
37.a. ranksb. achievesc. takesd. offers
38.a. wayb. spotc. lined. rate
(39-43). Read the passage below and then decided whether the statements that follow are True or False.

THE TELEPHONE

You may use the telephone every day but how much do you know about it? The telephone was invented by Alexander Bell in 1876. Bell was born in Scotland in 1847. Later he went to live in the USA. Bell was always interested

in sound. He wanted to be able to send sound through a wire. He had a workshop in his house in America and did many experiences there.

One day, he was doing an experiment in his workshop. He was careless and spilt some burning liquid onto his clothes. Talking into his telephone, Bell said, ‘Mr. Watson, I want you to come over here immediately, please.’ His assistant, Watson, was in another room far away from the workshop. However he heard Bell clearly on his own telephone. Quickly, he ran to Bell’s workshop. ‘Mr. Bell, I heard every word you said!’ Watson shouted excitedly.

Bell had finally succeeded. He had invented the first telephone. Later other inventors made better ones.

39. Alexander Bell invented the telephone when he was twenty six.

40. Bell emigrated from Scotland to the USA.

41. Bell did the one experiment and he succeeded.

42. Bell invented the telephone by chance.

43. Later the telephone was improved.

IV. WRITING

(44-46). Choose the sentence (a, b, c or d) that is almost the same in meaning as the sentence given.


44. He won’t find a seat unless he’s got a ticket.

a. He has got a ticket, and so will find a seat.

b. He can’t find a seat although he has a ticket.

c. He will be able to get a ticket if he finds a seat.

d. He will only get a seat if he has a ticket.

45. ‘You oughtn’t to drive fast.’ Jack’s mother told him.

a. Jack’s mother begged him not to drive fast.

b. Jack’s mother made him not to drive fast.

c. Jack’s mother advised him not to drive fast.

d. Jack’s mother suggested not driving fast.

46. It’s been fourteen years since I last saw my uncle.

a. I didn’t see my uncle fourteen years ago.

b. I see my uncle once every fourteen years.

c. I haven’t seen my uncle for fourteen years.

d. I saw my uncle when I was fourteen years old.

(47-50). Write complete sentences using the suggested words.

47. the air/ now/ polluted/ heavily/ traffic fumes//

______________________________________________

48. these math problems/ difficult/ us/ find/ answer//

______________________________________________

49. this/ house/ I/ born.

______________________________________________

50. I/ interested/ learn/ English/ and want/ improve/ speaking skill

______________________________________________

I.

1. a2. a3. b4. c5. d
II.

6. d7. a8. c9. a10. a
11. d12. d13. d14. b15. c
16. a17. b18. a19. a20. b


21. B (to seeing)22. A (which)23. B (interesting)
24. B (complained)25. D (she was)
26. informative27. representatives28. beautifully
29. talented30. difference
III.

31. c32. d33. a34. b35. d
36. c37. a38. b39. F40. T
41. F42. T43. T
IV.

44. d45. c46. c
47. The air is now heavily polluted with traffic fumes.

48. These math problems were difficult for us to find the answer.

49. This is the house where I was born.

50. I am interested in learning English and want to improve my speaking skill.

ĐỀ 29

I. PHONETICS

(1-3). Choose the word whose underlined part pronounced differently from that of the others.


1.a. proudb. youngc. foundd. out
2.a. schoolb. Christmasc. scholarshipd. chopsticks
3.a. deafb. headc. breadd. meat
(4-5). Choose the word whose main stress pattern is not the same as that of the others.

4.a. commonb. rubbishc. machined. cyclone
5.a. animalb. bacteriac. dynamited. pyramid
II. GRAMMAR AND VOCABULARY

(6-20). Choose the word or phrase (a, b, c or d) that best fits the blank space in each sentence.


6. I had no money____ me when I came across a nice shirt.

a. byb. atc. ond. over
7. Look_____ ! There’s a big hole in front of you

a. onb. overc. offd. out
8. She often goes swimming ______ Sunday mornings.

a. onb. overc. ind. at
9. Jack insisted that he didn’t need any help. I helped him anyway.

a. andb. soc. ford. but
10. Thousands of people took part in a _____ of support for free higher education.

a. marchb. gatheringc. demonstrationd. crowd
11. It is estimated that four million ______ watched the show on television.

a. observersb. onlookersc. viewersd. spectators
12. Some English words have the same pronunciation ___________ they are spelled differently, for example, dear and deer.

a. unlessb. sincec. even thoughd. only if
13. If the engine gets too hot, it________ to smoke.

a. startsb. is startingc. would startd. will be started
14. Do you know the man_______ over there?

a. standsb. who standc. stoodd. standing
15. I hope the children soon got used_______ in much smaller house.

a. liveb. to livec. livingd. to living
16. Help is needed for families________ homes were destroyed in the storm.

a. who’sb. whosec. whichd. whom
17. The seeds____ into flour which is used for pancakes, noodles and breads.

a. grindedb. are groundc. are groundedd. ground
18. _____the gold medal, he will have to do better than that.

a. To winb. So he winsc. That he winsd. Winning
19. When I first met him, he wore__________

a. a brown nice leather jacketb. a nice leather brown jacket
c. a leather nice brown jacketd. a nice brown leather jacket
20. ‘Won’t you have something to drink?’ ‘_________ ’

a. I’m sorry, I won’t.b. Not for me, thank you.
c. I’m afraid I can’t.d. I’ve no idea.
(21-25). Choose the underlined word or phrase (A, B, C or D) that needs correcting.

21. She cried (A) very hardly (B) when she heard (C) the news of (D) the accident.

22. It has been (A) a (B) long time when (C) I last wrote to (D) you.

23. The food that (A) my mother is cooking (B) in the kitchen is smelling (C) delicious (D).

24. We found some (A) garden furnitures (B) in that (C) old house we bought (D).

25. I know little (A) English, so (B) I’ll have this letter to translate (C) into (D) Vietnamese

(26-30). Use the correct form of the word given in each sentence.

26. The heating switches off _____________ . (automatic)

27. Malaysia has____________ climate, (tropic)

28. You should do what the_______________ told if you want to pass the driving test, (instruct)

29. He is a tourist guide so we all take his__________________ . (guide)

30. Your exam results are rather___________ . I expect you to do better, (disappoint)

READING

(31-38). Choose the word or phrase (a, b, c or d) that best fits the blank space in the following passage.


A 9.0 – 9.1 magnitude (31)_______ followed by a tsunami hit the east coast of Japan in March 11, 2011. With an underwater depth of 29 km, this was the largest earthquake to ever (32)_______ Japan in recorded history. Documented as the 4th most powerful earthquake in the world, it was (33)_________ for the death of 15,894 people with 6,152 injured and 2,562 people (34)________ . It also affected 20 prefectures, with over 127,000 buildings (35)________ destroyed and over 272,000 buildings “half collapsed”. The earthquake also caused a near nuclear (36)___________ when there was a partial meltdown in 3 (37)__________ of the Fukushima Daiichi nuclear power plant, (38)____ is the 2nd largest nuclear disaster after Chernobyl.

31.a. hurricaneb. earthquakec. typhoond. cyclone
32.a. occurb. shakec. striked. destroy
33.a. resultedb. sensiblec. reliabled. responsible
34.a. missingb. crossingc. passingd. including
35.a. stronglyb. fullyc. completelyd. frequently
36.a. energyb. disasterc. weapond. waste
37.a. contributorsb. controllersc. stationersd. reactors
38.a. whichb. thatc. whered. it
(39-43). Read the passage below carefully and choose the correct answer a, b, c or d.

MY HOME TOWN

I was born in Newcastle, a city in the North East of England. Newcastle is on the bank of the River Tyne. It is a quite big city of about 200,000 inhabitants. There is a cathedral and a university. There are five bridges over the River Tyne, which link Newcastle to the next town, Gateshead, where there is one of the biggest shopping centres in the world, the Metro Centre. ,

A few years ago, the main industries were shipbuilding and coal - mining, but now the chemical and soap industries are important.

I moved from Newcastle ten years ago but I often return. I miss the people, who are so warm and friendly, and I miss the wild, beautiful countryside near the city, where there are so many hills and streams.

39. Newcastle is ___________________ .

a. a city near the North East of England

b. a city in the North East of England

c. a city in the North of England

d. a small town in England

40. Newcastle has________________________ .

a. one of the biggest shopping centers

b. a wild, beautiful countryside

c. a population of about 200,000 people

d. one bridge that links it to the next town

41. Gateshead has one of_______________ in the world.

a. the largest rivers

b. the most important shipbuilding industries

c. the most beautiful countrysides

d. the biggest shopping centres

42. According to the passage, the writer_____________ .

a. is still living in Newcastle

b. has never returned to Newcastle

c. doesn’t live in Newcastle any more

d. has come back to live in Newcastle

43. Which of the following is NOT true about Newcastle?

a. Its people is friendly.

b. Its main industry now is shipbuilding,

c. It has a cathedral and a university,

d. It is next to Gateshead

WRITING

(44-46). Choose the sentence (a, b, c or d) that is almost the same in meaning as the sentence given.


44. You would like London if you visited it.

a. You’re unlikely to visit London.

b. You’ve never visited London,

c. You’re going to visit London soon.

d. You’ve already visited London.

45. Unlike his sister, Bob exercises every day.

a. Bob and his sister exercise every day.

b. Bob exercises every day, but his sister doesn’t.

c. Bob’s sister exercises every day, but he doesn’t.

d. Bob’s sister doesn’t exercise every day, and neither does he.

46. The bread was so stale to eat.

a. It was stale ‘to eat the bread

b. We cannot eat the bread because it was “burn.

c. Eating the bread was stale.

d. The bread was not fresh enough to eat.

(47-50). Rewrite the sentences so that they are nearest in meaning to the sentence printed before them.

47. Who will take care of the children when you are away?

Who will look_____________________________________________________

48. John could not find the way to the hotel.

John was not_____________________________________________________

49. The door was so heavy that the child could not push it open.

The door was too__________________________________________________

50. It’s ages since we met Lopez.

We______________________________________________________________

Đáp án

I.

1. b2. d3. d4. c5. b
II.

6. c7. d8. a9. d10. c
11. c12. c13. a14. d15. d
16. b17. b18. a19. d20. b


21. B (hard)22. A ( since)23. B (smells)
24. B (furniture)25. D (translated)
26. automatically27. tropical28.instructor
29. guidance30. disappointing
III.

31. b32. c33. d34. a35. c
36. b37. d38. a39. b40. c
41. d42. c43. b
IV.

44. b45. b46. d
47. Who will look after the children when you are away?

48. John was not able to find the way to the hotel.

49. The door was too heavy for the child to push it open.

50. We haven’t met Lopez for ages.

ĐỀ 30

I. PHONETICS

(1-3). Choose the word whose underlined part pronounced differently from that of the others.


1. a. authorb. ethnicc. healthd. gather
2. a. flewb. drewc. knewd. grew
3. a. celebrateb. capturec. conserved. comprise
(4-5). Choose the word whose main stress pattern is not the same as that of the others.

4. a. writerb. teacherc. builderd. career
5. a. companyb. atmospherec. customerd. employment
GRAMMAR AND VOCABULARY

(6-20). Choose the word or phrase (a, b, c or d) that best fits the blank space in each sentence.


6. Those boys are addicted_______ computer games.

a. onb. forc. tod. with
7. They will finish the work

a. onb. overc. ind. out
8. The design and material used for men were different___ __those used for women.

a. atb. fromc. withd. in
9. We have to start early______ we won’t be late.

a. so thatb. becausec. althoughd. otherwise
10. Will you buy an electric car when they_______ available?

a. becomeb. becamec. are becomingd. will become
11. Everybody must take part in________.

a. protectingb. preservingc. controllingd. preventing
12. Neil Armstrong,________ first walked in the moon, lived in the USA.

a. whob. thatc. whomd. whose
13. Baird produced the first TV picture in 1926,_________ ?

a. didn’t heb. wasn’t hec. doesn’t hed. hasn’t he
14. Traditionally, the ao dai_________ by both men and women.

a. frequently woreb. was frequent wornc. worn frequentlyd. was frequently worn
15. We______wear helmets in order to keep the traffic law and protect ourselves.

a. mayb. canc. shouldd. need
16. She asked me if I ______ any other language.

a. speakb. spokec. can speakd. would speak
17. Internet bars mustn’t let anybody_______ bad things.

a.b. to watchc. watchingd. watches
18. Would you be______ to hold the door open?

a. too kindb. so kindc. kind enoughd. as kind
19. It’s a pity you live so far away. I wish you________ .

a. lived nearerb. live nearc. lived neard. live nearer
20. Tm really sorry about that!’_______ ‘_____ !’

a. That’s rightb. Of coursec. You’re welcomed. It’s OK
(21-25). Choose the underlined word or phrase (A, B, C or D) that needs correcting.

21. Should I (A) be out when you call, just leave (B) a message with (C) my assistant? (D)

22. If I have many (A) homework (B) to do, I will not be (C) able to attend (D) the meeting.

23. When I arrived (A) home, my father was reading (B) the newspaper and (C) my mother watched (D) TV.

24. Most of (A) their farm (B) work used to do (C) by hand (D).

25. In the (A) United States, Mother’s (B) Day is celebrated (C) on second (D) Sunday in May.

(26-30). Use the correct form of the word given in each sentence.

26. It was completely_______because of poor planning, (fail)

27. Reduce means not buying products which are_______. (package)

28. We find the rice-cooking contest________. (interest)

29. The main source of income for most newspapers is_______ advertising, (commerce)

30. Computers were a wonderful_______at the time, (invent)

READING

(31-38). Choose the word or phrase (a, b, c or d) that best fits the blank space in the following passage.


Every child in Great Britain between the age of five and fifteen must 31)____ school. There are three main types of educational institutions: primary (elementary) schools, (32)____ schools, and universities. State schools are free, and attendance is (33)________ . Morning school begins at nine o’clock and (34)______ until half past four. School is open five days a week. On Saturdays and Sundays there are (35)______ lessons. There are holidays at Christmas, Easter and in summer. In London as in all cities there are two grades of state schools for (36)______ who will go to work at fifteen: primary schools for boys and girls (37)______ the ages of five and eleven, and secondary schools for children from eleven to fifteen years. The lessons are: reading, writing, the English language, English literature, English history, geography, science, (38)______ study, drawing, painting, singing, woodwork and drill (physical training).

31. a. attendb. presentc. visitd. leave
32. a. privateb. preparatoryc. secondaryd. nursery
33. a. freeb. compulsoryc. regulard. important
34. a. endsb. finishesc. lastsd. passes
35. a. someb. nonec. notd. no
36. a. thisb. thesec. thatd. those
37. a. inb. betweenc. fromd. of
38. a. Natureb. Naturalc. Naturalizedd. Natured
(39-43). Read the passage below and then decided whether the statements that follow are True or False.

Lunar New Year, or Tet, is Vietnam’s main holiday. It is the biggest and the most important occasion in the year which falls sometime between 19th January and 20 February on the Western calendar.

Tet preparations and celebrations used to be spread over months, but nowadays the holiday is much shorter. However, a great deal of excitement still builds up well before Tet. Shops are full of goods. People are busy buying gifts, cleaning and decorating their houses and cooking traditional foods such as bank chung, bank tet.

On the days of Tet, people visit other family members and friends, and they exchange New Year’s wishes. Children receive their “lucky money” inside red envelopes. Both children and adults take part in games and various forms of entertainment. Tet is really a time of fun and festivals throughout the country.

39. Tet or Lunar New Year holiday is the most important celebration for Vietnamese people.

40. Tet is always on 20th February on the Western calendar.

41. Nowadays, Tet lasts longer than it used to be.

42. According to the text, “lucky money” is given to everyone at Tet.

43. Tet is a great occasion of joy and of entertainment.

WRITING

(44-46). Choose the best sentence (a, b, c or d) made from the given cues.


44. cars/ fast and comfortable/ than/ motorcycles//

a. Cars are faster and more comfortable than motorcycles.

b. Cars are more fast and comfortable than motorcycles are.

c. Cars are faster and comfortable than motorcycles are.

d. Cars are fast and comfortable than motorcycles.’

45. how/ spend/ homework?//

a. How much time you spend on doing your homework?

b. How long do you spend to do your homework?

c. How much time do you spend oil your homework?

d. How do you spend your time in your homework?

46. The children/ enjoy/ take/ Dam Sen Water Park//

a. The children enjoy taking to Dam Sen Water Park.

b. The children enjoy to take to Dam Sen Water Park.

c. The children enjoy being taken to Dam Sen Water Park.

d. The children enjoy to be taken to Dam Sen Water Park.

(47-50). Rewrite the sentences so that they are nearest in meaning to the sentence printed before them.

47. Please don’t play your music so loudly.

Would you mind________________________________________________ ?

48. Let’s go camping tomorrow.

Why don’t_____________________________________________________ ?

49. Children shouldn’t swim in this pool because it is too deep.

This pool is not_________________________________________________

50. The house is really beautiful!

What__________________________________________________________!

Đáp án

I.

1. d2. c3.4. d5. d
II.

6. c7. a8. b9. a10. a
11. d12. a13. a14. d15. c
16. b17. a18. c19. a20. d


21. D (.)22. A (much)23. D (was watching)
24. C (to be done)25. D (the second)
26. failure27. overpackaged28. interesting
29. commercial30. invention
III.

31. a32. c33. b34. c35. d
36. d37. b38. a
39. T40. F41. F42. F43. T
IV.

44. a45. c46. c
47. Would you mind not playing your music so loudly?

48. Why don’t we go camping tomorrow?

49. This pool is no shallow enough for children to swim in.

50. What a beautiful house!

Mời bạn đọc tham khảo thêm nhiều tài liệu Ôn thi vào lớp 10 năm học 2023 - 2024 khác tại đây:

https://yopo.vn/forums/de-thi-vao-lop-10.218/

1684407292115.png
 

DOWNLOAD FILE

  • YOPOVN.COM---Sách bộ 30 đề TS lớp 10 NH 23-24.doc
    765 KB · Lượt xem: 7
Nếu bạn cảm thấy nội dung chủ đề bổ ích , Hãy LIKE hoặc bình luận để chủ đề được sôi nổi hơn
  • Từ khóa
    10 đề thi tiếng anh vào 10 35 đề tiếng anh thi vào lớp 10 word bài tập ôn thi vào lớp 10 môn tiếng anh bộ đề ôn thi vào 10 môn tiếng anh (with key) các đề ôn tập tiếng anh vào lớp 10 de thi tiếng anh vào 10 các tỉnh file đề thi tiếng anh vào 10 file đề thi tiếng anh vào 10 hà nội 2021 file đề thi vào 10 môn tiếng anh giải ôn tập thi vào lớp 10 môn tiếng anh giải đề thi tiếng anh vào 10 hà nội hướng dẫn ôn tập thi vào 10 môn tiếng anh hướng dẫn ôn tập thi vào 10 tiếng anh một số đề thi tiếng anh vào 10 ôn tập thi tuyển sinh vào 10 môn tiếng anh ôn tập thi vào lớp 10 môn tiếng anh 2019 ôn tập thi vào lớp 10 môn tiếng anh violet ôn tập thi vào lớp 10 tiếng anh ôn tập tiếng anh vào 10 ôn tập tiếng anh vào 10 nguyễn thị chi ôn tập tiếng anh vào lớp 10 ôn tập vào 10 tiếng anh ôn thi lớp 10 môn tiếng anh có đáp án ôn thi lớp 10 tiếng anh ôn tiếng anh vào lớp 10 sách ôn tập vào 10 môn tiếng anh sách ôn tập vào 10 tiếng anh đề cương ôn tập tiếng anh thi vào 10 đề cương ôn tập vào lớp 10 môn tiếng anh đề thi môn tiếng anh vào 10 năm 2019 đề thi môn tiếng anh vào 10 năm 2020 đề thi môn tiếng anh vào 10 năm 2021 đề thi thử tiếng anh vào 10 có đáp án đề thi thử tiếng anh vào 10 mới nhất đề thi tiếng anh tuyển sinh vào 10 đề thi tiếng anh vào 10 đề thi tiếng anh vào 10 2020 đề thi tiếng anh vào 10 2021 đề thi tiếng anh vào 10 an giang 2021 đề thi tiếng anh vào 10 bắc giang đề thi tiếng anh vào 10 bắc giang 2020 đề thi tiếng anh vào 10 bắc giang 2021 đề thi tiếng anh vào 10 bắc kạn đề thi tiếng anh vào 10 bắc ninh 2020 đề thi tiếng anh vào 10 bắc ninh 2021 đề thi tiếng anh vào 10 bình dương đề thi tiếng anh vào 10 bình định đề thi tiếng anh vào 10 bình định 2019 đề thi tiếng anh vào 10 bình định 2020 đề thi tiếng anh vào 10 bình định năm 2018 đề thi tiếng anh vào 10 các năm đề thi tiếng anh vào 10 chu văn an đề thi tiếng anh vào 10 chuyên đề thi tiếng anh vào 10 chuyên chu văn an đề thi tiếng anh vào 10 chuyên ngoại ngữ đề thi tiếng anh vào 10 chuyên sư phạm đề thi tiếng anh vào 10 có file nghe đề thi tiếng anh vào 10 có đáp án đề thi tiếng anh vào 10 có đáp an hà nội đề thi tiếng anh vào 10 có đáp án violet đề thi tiếng anh vào 10 file word đề thi tiếng anh vào 10 full trắc nghiệm đề thi tiếng anh vào 10 hà nội đề thi tiếng anh vào 10 hà nội 2018 đề thi tiếng anh vào 10 hà nội 2019 đề thi tiếng anh vào 10 hà nội 2020 đề thi tiếng anh vào 10 hà nội 2021 đề thi tiếng anh vào 10 hà nội 2021 mã đề 119 đề thi tiếng anh vào 10 hải dương 2017 đề thi tiếng anh vào 10 hải dương 2018 đề thi tiếng anh vào 10 hải dương 2019 đề thi tiếng anh vào 10 hải dương 2020 đề thi tiếng anh vào 10 hải dương 2021 đề thi tiếng anh vào 10 hải phòng 2020 đề thi tiếng anh vào 10 hải phòng 2021 đề thi tiếng anh vào 10 hồ chí minh đề thi tiếng anh vào 10 khánh hòa 2021 đề thi tiếng anh vào 10 không chuyên đề thi tiếng anh vào 10 không đáp án đề thi tiếng anh vào 10 kiên giang đề thi tiếng anh vào 10 lạng sơn đề thi tiếng anh vào 10 lạng sơn 2020 đề thi tiếng anh vào 10 lào cai đề thi tiếng anh vào 10 lào cai 2018 đề thi tiếng anh vào 10 lào cai 2019 đề thi tiếng anh vào 10 lào cai 2020 đề thi tiếng anh vào 10 lào cai 2021 đề thi tiếng anh vào 10 lương thế vinh đề thi tiếng anh vào 10 mã 001 đề thi tiếng anh vào 10 mới nhất đề thi tiếng anh vào 10 môn tiếng anh đề thi tiếng anh vào 10 năm 2018 đề thi tiếng anh vào 10 năm 2019 đề thi tiếng anh vào 10 năm 2020 đề thi tiếng anh vào 10 năm 2020 hà nội đề thi tiếng anh vào 10 năm 2021 đề thi tiếng anh vào 10 năm 2021 hà nội đề thi tiếng anh vào 10 nam định đề thi tiếng anh vào 10 nghệ an đề thi tiếng anh vào 10 nghệ an 2019 đề thi tiếng anh vào 10 nghệ an 2020 đề thi tiếng anh vào 10 ở hà nội đề thi tiếng anh vào 10 online đề thi tiếng anh vào 10 pdf đề thi tiếng anh vào 10 phú thọ đề thi tiếng anh vào 10 phú thọ 2017 đề thi tiếng anh vào 10 phú thọ 2018 đề thi tiếng anh vào 10 phú thọ 2019 đề thi tiếng anh vào 10 phú thọ 2020 đề thi tiếng anh vào 10 phú yên 2021 đề thi tiếng anh vào 10 quảng ngãi đề thi tiếng anh vào 10 quảng ngãi 2020 đề thi tiếng anh vào 10 quảng ngãi 2021 đề thi tiếng anh vào 10 quảng ninh đề thi tiếng anh vào 10 quảng ninh 2018 đề thi tiếng anh vào 10 quảng ninh 2019 đề thi tiếng anh vào 10 quảng ninh 2020 đề thi tiếng anh vào 10 quảng ninh 2021 đề thi tiếng anh vào 10 quốc gia 2020 đề thi tiếng anh vào 10 sở hà nội đề thi tiếng anh vào 10 thái bình đề thi tiếng anh vào 10 thái nguyên đề thi tiếng anh vào 10 thanh hóa đề thi tiếng anh vào 10 thanh hóa 2019 đề thi tiếng anh vào 10 thanh hóa 2020 đề thi tiếng anh vào 10 tỉnh phú thọ đề thi tiếng anh vào 10 tỉnh quảng ninh đề thi tiếng anh vào 10 trắc nghiệm đề thi tiếng anh vào 10 và đáp án đề thi tiếng anh vào 10 vietjack đề thi tiếng anh vào 10 vĩnh phúc đề thi tiếng anh vào 10 vĩnh phúc 2019 đề thi tiếng anh vào 10 vĩnh phúc 2020 đề thi tiếng anh vào 10 vĩnh phúc 2021 đề thi tiếng anh vào 10 violet đề thi tiếng anh vào 10 vndoc đề thi tiếng anh vào 10 vũng tàu đề thi tiếng anh vào 10 yên bái đề thi tiếng anh vào lớp 10 đề thi tiếng anh vào lớp 10 2020 đề thi tiếng anh vào lớp 10 2021 đề thi tiếng anh vào lớp 10 amsterdam đề thi tiếng anh vào lớp 10 an giang đề thi tiếng anh vào lớp 10 chuyên hưng yên đề thi tiếng anh vào lớp 10 chuyên sư phạm đề thi tiếng anh vào lớp 10 có file nghe đề thi tiếng anh vào lớp 10 daklak đề thi tiếng anh vào lớp 10 dễ đề thi tiếng anh vào lớp 10 khánh hòa đề thi tiếng anh vào lớp 10 khánh hòa 2019 đề thi tiếng anh vào lớp 10 kon tum đề thi tiếng anh vào lớp 10 ở hà nội đề thi tiếng anh vào lớp 10 pdf đề thi tiếng anh vào lớp 10 phú yên đề thi tiếng anh vào lớp 10 tiền giang đề thi tiếng anh vào lớp 10 tỉnh hưng yên
  • HỖ TRỢ ĐĂNG KÝ VIP

    Liên hệ ZALO để được tư vấn, hỗ trợ: ĐĂNG KÝ TÀI KHOẢN VIP
    ZALO:0979702422

    BÀI VIẾT MỚI

    Thống kê

    Chủ đề
    35,706
    Bài viết
    37,174
    Thành viên
    138,499
    Thành viên mới nhất
    UFYBBGYUDT

    BQT trực tuyến

    • Yopovn
      Ban quản trị Team YOPO
    Top
    CHỈ THÀNH VIÊN VIP MỚI ẨN ĐƯỢC QUẢNG CÁO!

    Chúng tôi hiểu rồi, quảng cáo thật khó chịu!

    Chắc chắn, phần mềm chặn quảng cáo đã loại bỏ quảng cáo của diễn đàn. Điều này là không được phép đối với người dùng truy cập. Chỉ THÀNH VIÊN VIP mới có thể ẩn tính năng quảng cáo.

    XEM THƯ MỤC MIỄN PHÍ❌❌DANH SÁCH THƯ MỤC MIỄN PHÍ UPDATE 2024!!!Hãy tắt chức năng ẩn quảng cáo từ trình duyệt của bạn để tiếp tục sử dụng chức năng của website.

    ƯU ĐÃI ƯU ĐÃI THÀNH VIÊN VIP ⏱ DỊP 30/4 - 1/5
    ĐÃ TẮT CHẶN QUẢNG CÁO    KHÔNG. CẢM ƠN!